Patho Final Exam

Pataasin ang iyong marka sa homework at exams ngayon gamit ang Quizwiz!

A 40-year-old male undergoes surgery for a PTH-secreting tumor in which the parathyroid is removed. What would the nurse expect following surgery

decrease calcium reabsorption in the kidney

Elevated levels of glucocorticoids result in which of the following assessment findings (Select all that apply.) Polycythemia Increased appetite Weight loss Decreased calcium Increased height

decreased calcium increased appetite polycythemia

A 40-year-old male undergoes surgery for a PTH-secreting tumor in which the parathyroid is removed. Which of the following would the nurse expect following surgery Increased serum calcium Decreased bone formation Decreased calcium reabsorption in the kidney Increased calcitonin

decreased calcium reabsorption in the kidney

A 19-year-old female with type 1 DM was admitted to the hospital with altered consciousness and the following lab values: serum glucose 500 mg/dl (high) and serum K 2 )low. Her parents state that she has been sick with the "flu" for a week. THe diagnosis is hyperolsmolar hyperglycemiz nonketotic syndrome. What relationship do these values have with her insulin deficiency?

decreased insulin causes hyperglyceima and osmotic diuresis

Status epilepticus is considered a medical emergency because of the:

development of cerebral hypoxia.

Palpation of the neck of a patient diagnosed with Graves disease would most likely reveal:

diffuse thyroid enlargement.

A 25-year-old female is diagnosed with urinary tract obstruction. While planning care, the nurse realizes that the patient is expected to have hydronephrosis and a decreased glomerular filtration rate caused by:

dilation of the renal pelvis and calyces proximal to a blockage.

When insulin binds to its receptors on muscle cells, an increase in glucose uptake by the muscle cells occurs. This is an example of a _effect by a hormone. Pharmacologic Permissive Biphasic Direct

direct

Clinical manifestations of Parkinson disease are caused by a deficit in which neurotransmitter?

dopamine

A patient has high levels of hormones. To adapt to the high hormone concentrations, the patient's target cells have the capacity for: Negative feedback Positive feedback Down-regulation Up-regulation

down-regulation

What is the cause of the hyperpigmentation seen in people with Cushing syndrome?

elevated levels of ACTH

the body's inability to conserve water and sodium when affected by Addison disease is explained by what condition?

elevated levels of cortisol

An endocrinologist isolated a new hormone and found it to be a water-soluble amine. Which of the following is most like this new hormone Growth hormone (GH) Luteinizing hormone (LH) Antidiuretic hormone (ADH) Epinephrine

epinephrine

An endocrinologist isolated a new hormone and found it to be a water-soluble amine. Which of the following is most like this new hormone?

epinephrine

If a patient had a problem with the adrenal medulla, what hormone should the nurse monitor?

epinephrine

If a patient had a problem with the adrenal medulla, which of the following hormones should the nurse monitor Cortisol Epinephrine Androgens Estrogens

epinephrine

The most common clinical manifestation of portal hypertension is _____ bleeding.

esophageal

Which of the following are characteristic of idiopathic pulmonary arterial hypertension (IPAH)?

fatigue, dyspnea, and jugular vein distention

Cirrhosis causes intrahepatic portal hypertension in children as a result of:

fibrosis increasing resistance to blood flow within the portal system.

A 30-year-old male is diagnosed with a hormone-secreting tumor of the pancreas alpha cells. Which of the following would the nurse expect to be most likely increased in this patient Amylin Glucagon Insulin Somatostatin

glucagon

Frank bleeding of the rectum is called:

hematochezia Hematochezia is frank bright red or burgundy blood from the rectum. (p. 1456)

While checking the lab results for a patient with Graves disease, the nurse would check the T3 level to be abnormally:

high

Which principle should the nurse include while planning care for a patient with an ADH problem? ADH release from the posterior pituitary is stimulated by:

high serum osmolarity sensed by osmoreceptors in the hypothalamus

While checking the lab results for a patient diagnosed with Graves disease, the nurse would expect the T3 level to be abnormally:

high.

When a staff member asks the nurse what causes the chronic complications of DM such as microvascular and macrovasular disease, how should the nurse respond? These complications are primarily related to:

hyperglycemia

When catecholamines are released in a patient, what should the nurse assess for Nutrient absorption Fluid retention Hypotension Hyperglycemia

hyperglycemia

When catecholamines are released in a patient, what should the nurse assess for?

hyperglycemia

Lois is a 56 year old woman who has been diagnosed with Type 2 diabetes. She is being treated with 1800 calorie diet, exercise, glucophage 500 mg po bid ( oral hypoglycemic agent) and 2 units of NPH at night. She has a history of difficulty in maintaining a tight control of the glucose levels as indicated by high levels of Hg A-1c. She goes on a 5 day Caribbean cruise. The day after she returned from the cruise, her husband brought her to the ER stating that she was lethargic and confused. He indicates that her blood sugar was "too high to read". The hospital Lab values included: blood sugar 800, BUN 25, Cr 1.1, glucose was negative for ketones. Arterial blood gas (ABG) indicates metabolic acidosis. BP was 104/60 ( her normal is 150/80), HR 110, RR 18 and deep. She is a woman approximately 5'8'' and weighs about 180 lbs. Which of the following are problems/complications in this case?

hyperglycemic hyperosmolar nonketotic syndrome and dehydration

A patient with Addison disease has weakness and easy fatigability. A nurse recalls this is due to:

hypocortisolism

The nurse assesses a patient for short term complications of diabetes. This assessment includes evaluations for:.

hypoglycemia,hyperglycemia and ketoacidosis.

A 50-year-old female presents with lightheadedness and overall abnormal feeling. Hyperaldosteronism is diagnosed. What symptoms would the nurse expect?

hypokalemia

A 54-year-old patient with pulmonary tuberculosis is evaluated for syndrome of inappropriate ADH secreation. What electrolyte imbalances would be expected in this patient?

hyponatremia

When the endocrinologist asks the staff how the releasing hormones that are made in the hypothalamus travel to the anterior pituitary, how should the staff replay? Via the:

hypophysial portal system

When the endocrinologist asks the staff how the releasing hormones that are made in the hypothalamus travel to the anterior pituitary, how should the staff reply Via the: Vessels of the zona fasciculata Chromophils Median eminence Hypophysial portal system

hypophysial portal system

Anemia accompanies chronic renal failure because of:

inadequate production of erythropoietin.

A nurse is reviewing lab results. What lab results would slow down the rate of parathyroid hormone secretion?

increase serum calcium levels

A nurse is reviewing lab results. Which of the following lab results would slow down the rate of parathyroid hormone secretion Increased serum calcium levels Decreased serum calcium levels Decreased levels of TSH Increased levels of TSH

increased serum calcium levels

A 50-year-old male patient is deficient in ADH production. Which of the following assessment findings would the nurse expect to find Increased blood volume Increased urine osmolality Increased urine volume Increased arterial vasoconstriction

increased urine volume

A 70-year-old female has brittle bones secondary to osteoporosis. Her primary care provider prescribes calcitonin to:

inhibit calcium resorption from bones

A 70-year-old female has brittle bones secondary to osteoporosis. Her primary care provider prescribes calcitonin to: Activate vitamin D Stimulate osteoclastic activity Inhibit calcium resorption from bones Promote thyroid hormone release

inhibit calcium resorption from bones

A nurse is teaching staff about protein hormones. What information should the nurse include? One of the protein hormones is:

insulin

A nurse is teaching staff about protein hormones. Which information should the nurse include One of the protein hormones is: Thyroxine (T4) Aldosterone Testosterone Insulin

insulin

A nurse is teaching staff about protein hormones. Which information should the nurse include? One of the protein hormones is:

insulin

An aide asks the nurse what activates tyrosine. What is the nurse's best response GH PRL Insulin Estrogen

insulin

A 19-year-old female with type 1 DM was admitted to the hospital with the following lab values: serum glucose 500 mg/dl (high), urine glucose and ketones 4+ (high), and arterial pH 7.2 (low). Her parents state that she has been sick with the "flu" for a week. What statement best explains her acidotic state?

insulin deficiency promotes lipid metabolism and ketone formation

Which nutrient would the nurse encourage the patient to consume for thyroid hormone synthesis Zinc Sodium Iodine Calcium

iodine

Which nutrient would the nurse encourage the patient to consume for thyroid hormone synthesis?

iodine

A peptic ulcer may occur in all of the following areas except the:

jejunum.

A 30-year-old female with Graves disease is admitted to a hospital unit for the surgical removal of her thyroid gland. During the postoperative period the nurse notes that the patient's serum calcium is low. The nurse should observe that patient for what sign and symptoms?

laryngeal spasms and hyperrelexia

An initial assessment finding associated with acute spinal cord injury is _____ the injury.

loss of voluntary control below

While planning care for a patient with hypothyroidism, what principle should the nurse remember? The basal metabolic rate is unusually ________ with hypothyroidism.

low

When considering hypothyroidism, the basal metabolic rate is unusually:

low.

If the patient has a problem with the pineal gland, which substance would the nurse monitor Melatonin Epinephrine Cortisol Somatostatin

melatonin

if a patient has a problem with the pineal gland, what substance would the nurse monitor?

melatonin

A 3-year-old male was diagnosed with congenital hypothyroidism. The parents ask the nurse if left untreated what will happen. What is the nurse's best response? If left untreated the child would have:

mental retardation and stunted growth

The clinical manifestations of Parkinson disease (PD) include:

muscle tremor; slow, rigid movements; and postural abnormalities.

While planning care for a patient from general anesthesia, which principle should the nurse remember? A side effect of some general anesthetic agents is _____ diabetes insipidus.

nephrogenic

A nurse recalls direct stimulation of the insulin-secreting cells of the pancreas by the autonomic nervous system is an example of _control. Negative feedback Positive feedback Neural Substrate-level dependent

neural

A 30-year-old male was diagnosed with thyroid carcinoma. The lab tests the nurse would most likely find are _______ T3 and T4 levels:

normal

A patient diagnosed with thyroid carcinoma would be expected to have T3 and T4 levels that are:

normal.

Visual disturbances are a common occurrence in patients with untreated Graves disease. The endocrinologist explains to the patient that the main cause of these complications is:

orbital edema and protrusion of the eyeball

Visual disturbances are a common occurrence in patients with untreated Graves disease. The endocrinologist explains to the patient that the main cause of these complications is:

orbital edema and protrusion of the eyeball.

The type of diarrhea that is a result of unhydrolyzed lactose is referred to as:

osmotic

What problem should the nurse assess for in a patient with chronic hyperparatheyroidism?

osteoporosis and pathologic fractures

A 39-year-old female is recovering from the birth of her third child. Which hormone would help prevent uterine bleeding Aldosterone Cortisol Prolactin Oxytocin

oxytocin

If a patient had a problem with the hypothalamus, which of the following hormones would be affected ACTH Oxytocin ADH TSH

oxytocin

if a patient had a problem with the hypothalamus, what hormone would be affected

oxytocin

An 11-year-old male is newly diagnosed with type 1 DM. What classic symptoms should the nurse assess the patient for?

polydipsia, polyuria, polyphagia, and weight loss

When a staff member asks the nurse which gland secretes ADH and oxytocin, how should the nurse respond Anterior pituitary Posterior pituitary Hypothalamus Pineal gland

posterior pituitary

When a staff member asks the nurse which gland secretes ADH and oxytocin, how should the nurse respond?

posterior pituitary

A nurse recalls insulin has an effect on which of the following groups of electrolytes Sodium, chloride, phosphate Calcium, magnesium, potassium Hydrogen, bicarbonate, chloride Potassium, magnesium, phosphate

potassium, magnesium, phosphate

A 22-year-old female has a low level of TSH. What condition does the nurse expect the patient is experiencing?

secondary hypothyroidism

A 10-year-old female was brought to the ER following a sudden onset of convulsions. The primary care provider thinks that she experienced an explosive, disorderly discharge of cerebral neurons referred to as:

seizure

A sudden, explosive, disorderly discharge of cerebral neurons is termed:

seizure.

A nurse is teaching a patient about insulin. What information should the nurse include? Insulin is primary regulated by:

serum glucose levels

A nurse is teaching a patient about insulin. Which information should the nurse include Insulin is primarily regulated by: Metabolic rate Serum glucose levels Prostaglandins Enzyme activation

serum glucose levels

A 35-year-old female with Graves disease is admitted to a medical-surgical unit. What symptoms would the nurse expect to find before treatment?

skin hot and moist, rapid heart rate

A patient wants to know what can cause ACTH to be released. How should the nurse respond High serum levels of cortisol Hypotension Hypoglycemia Stress

stress

A patient wants to know what can cause ACTH to be released. How should the nurse respone?

stress

Which of the following patients is the most at risk for developing hypernatremia? A patient with: a. Vomiting b. Diuretic use c. Dehydration d. Hypoaldosteronism

c. Dehydration Rationale: Dehydration leads to hypernatremia because an increase in sodium occurs with a net loss in water. Vomiting leads to hyponatremia. Diuretic use would lead to sodium loss. Hypoaldosteronism leads to hyponatremia.

A nurse is assessing a patient with hypoparathyroidism. Clinical manifestations of hypoparathyroidism include: (select all that apply)

tetany, Chvostek sign, Trousseau sign, hair loss.

When a nurse is teaching about urinary pathogens in men. Which information should the nurse include? Mechanisms for defense against urinary pathogens in men include

the long length of the urethra and antimicrobial secretions from the prostate.

An ednocrinologist is teaching about aldosterone secretion. What information should the endocrinolgist include? Aldosterone secretion is regulated by:

the renin-angiotension system

An endocrinologist is teaching about aldosterone secretion. Which information should the endocrinologist include Aldosterone secretion is regulated by: The sympathetic nervous system ACTH feedback The renin-angiotension system Positive feedback

the renin-angiotension system

Diabetes insipidus, diabetes mellitus, and SIADH share what assessment manifestations?

thirst

More than _____ stools per day is considered abnormal.

three(3)

A 30-year-old male was diagnosed with hypothyroidism. Synthesis of which of the following would decrease in this patient Corticosteroid B globulin Sex hormone-binding globulin Thyroid-binding globulin Albumin

thyroid binding globulin

Mrs. Johnson is admitted to your unit with tachycardia, fever, agitation, and diarrhea. Her assessment revealed hyperplasia of the thyroid gland and elevated triiodo-L-thyronine (T3). She also reports that she has been very upset at the recent death of her mother. Her diagnosis is Graves disease and thyrotoxic crisis. Graves disease develops from a(n):

thyroid-stimulating immunoglobulins that causes overproduction of thyroid hormones.

While planning care for a patient with renal calculi, the nurse remembers the most important factor in renal calculus formation is:

urine pH.

A nurse is teaching staff about oxytocin. What information should the nurse include? Target cells for oxytocin are located in the:

uterus

A nurse is teaching the staff about oxytocin. Which information should the nurse include Target cells for oxytocin are located in the: Renal tubules Thymus Liver Uterus

uterus

Which patient is most prone to metabolic alkalosis? A patient with: a. Retention of metabolic acids b. Hypoaldosteronism c. Excessive loss of chloride (Cl) d. Hyperventilation

c. Excessive loss of chloride (Cl) Rationale: When acid loss is caused by vomiting, renal compensation is not very effective because loss of Cl stimulates renal retention of bicarbonate, leading to alkalosis. Retention of metabolic acids would lead to acidosis, not alkalosis. Hypoaldosteronism leads to hyponatremia and does not cause alkalosis. Hyperventilation leads to respiratory alkalosis, not metabolic alkalosis.

Which statement by the staff indicates teaching was successful concerning aldosterone? Secretion of aldosterone results in: a. Decreased plasma osmolality b. Increased serum potassium levels c. Increased blood volume d. Localized edema

c. Increased blood volume Rationale: Aldosterone promotes renal sodium and water reabsorption and excretion of potassium, thus, increasing blood volume.

A 70-year-old male with chronic renal failure presents with edema. Which of the following is the most likely cause of this condition? a. Increased capillary oncotic pressure b. Decreased interstitial oncotic pressure c. Increased capillary hydrostatic pressure d. Increased interstitial hydrostatic pressure

c. Increased capillary hydrostatic pressure Rationale: Increased capillary hydrostatic pressure would facilitate increased movement from the capillary to the interstitial space, leading to edema.

Which organ system should the nurse monitor when the patient has long-term potassium deficits? a. Central nervous system (CNS) b. Lungs c. Kidneys d. Gastrointestinal tract

c. Kidneys Rationale: Long-term potassium deficits lasting more than 1 month may damage renal tissue, with interstitial fibrosis and tubular atrophy.

A nurse recalls regulation of acid-base balance through removal or retention of volatile acids is accomplished by the: a. Buffer systems b. Kidneys c. Lungs d. Liver

c. Lungs Rationale: The volatile acid is carbonic acid (H2CO3), which readily dissociates into carbon dioxide (CO2) and water (H2O). The CO2 is then eliminated by the lungs.

A 60-year-old female is diagnosed with hyperkalemia. Which assessment finding should the nurse expect to observe? a. Weak pulse b. Excessive thirst c. Oliguria d. Constipation

c. Oliguria Rationale: Hyperkalemia is manifested by oliguria. Hypokalemia is manifested by a weak pulse; it is not caused by hyperkalemia. Hypokalemia is manifested by excessive thirst. Diarrhea, not constipation, is a manifestation of hyperkalemia.

Which finding would support the diagnosis of respiratory acidosis? a. Vomiting b. Hyperventilation c. Pneumonia d. An increase in noncarbonic acids

c. Pneumonia Rationale: Respiratory acidosis occurs with hypoventilation, and pneumonia leads to hypoventilation. Vomiting leads to loss of acids and then to alkalosis. Hyperventilation leads to respiratory alkalosis, not acidosis. Metabolic acidosis is caused by an increase in noncarbonic acids.

A 54-year-old male with a long history of smoking complains of excessive tiredness, shortness of breath, and overall ill feelings. Lab results reveal decreased pH, increased CO2, and normal bicarbonate ion. These findings help to confirm the diagnosis of: a. Respiratory alkalosis b. Metabolic acidosis c. Respiratory acidosis d. Metabolic alkalosis

c. Respiratory acidosis Rationale: A decreased pH indicates acidosis. With increased CO2, it is respiratory acidosis.

When a patient asks what the most common type of renal stones is composed of, how should the nurse respond? The most common type of renal stone is composed of:

calcium

When a patient asks what the most common type of renal stones is composed of, how should the nurse respond? The most common type of renal stone is composed of:

calcium.

What diseases should the nurse teach the patient to prevent as it is the ultimate cause of death in the patient with diabetes?

cardiovascular disease

When a patient asks about target cell receptors, what is the nurse;s best response? Target cell receptors for most water soluble hormones are located in the:

cell membrane

When a patient asks about target cell receptors, which is the nurse's best response Target cell receptors for most water-soluble hormones are located in the: Cytosol Cell membrane Endoplasmic reticulum Nucleus

cell membrane

Which of the following assessment findings would be expected in pulmonary embolism (PE)?

chest pain, tachycardia, tachypnea, fever, hemoptysis

A student asks the instructor which of the following is the most potent naturally occurring glucocorticoid. How should the instructor respond Aldosterone Testosterone Cortisol Prolactin

cortisol

What type of cyst develops when an ovarian follicle is stimulated but no dominant follicle develops and completes the maturity process? a. Follicular b. Corpus luteal c. Corpus albicans d. Benign ovarian

d. Benign ovarian

Which statement regarding pelvic inflammatory disease (PID) is true? a. An episode of mild PID can decrease the possibility of a successful pregnancy by 80%. b. Such an inflammation results in temporary changes to the ciliated epithelium of the fallopian tubes. c. PID has not been associated with an increased risk of an ectopic pregnancy. d. Contracting this infection increases the risk of uterine cancer.

d. Contracting this infection increases the risk of uterine cancer.

Which condition is considered a clinical cause of amenorrhea? a. Disorder in the endometrium b. Obstruction of the fallopian tubes c. Lack of physical exercise d. Failure to ovulate

d. Failure to ovulate

What is usually the first clinical manifestation of breast cancer? a. Nipple dimpling b. Nipple discharge c. Enlargement of one breast d. Painless lump

d. Painless lump

In 95% of children of delayed puberty, the problem is caused by: a. Disruption in the hypothalamus b. Disruption of the pituitary c. Deficit in estrogen or testosterone d. Physiologic hormonal delays

d. Physiologic hormonal delays

What is the leading cause of infertility in women? a. Pelvic inflammatory disease b. Endometriosis c. Salpingitis d. Polycystic ovary syndrome

d. Polycystic ovary syndrome

A 35-year-old male weighs 70 kg. Approximately how much of this weight is ICF? a. 5 L b. 10 L c. 28 L d. 42 L

d. 42 L

Which of the following patients is most prone to hypochloremia? A patient with: a. Hypernatremia b. Hypokalemia c. Hypercalcemia d. Increased bicarbonate intake

d. Increased bicarbonate intake Rationale: Hypochloremia is the result of elevated bicarbonate concentration, as occurs in metabolic alkalosis.

An experiment was designed to test the effects of the Starling forces on fluid movement. Which of the following alterations would result in fluid moving into the interstitial space? a. Increased capillary oncotic pressure b. Increased interstitial hydrostatic pressure c. Decreased capillary hydrostatic pressure d. Increased interstitial oncotic pressure

d. Increased interstitial oncotic pressure

A 5-year-old male presents to the ER with delirium and sunken eyes. After diagnosing him with severe dehydration, the primary care provider orders fluid replacement. The nurse administers a hypertonic intravenous solution. Which of the following would be expected? a. Symptoms subside quickly b. Increased ICF volume c. Decreased ECF volume d. Intracellular dehydration

d. Intracellular dehydration A hypertonic solution would cause fluid to move into the extracellular space, leading to intracellular dehydration.

A nurse is reviewing lab reports. The nurse recalls blood plasma is located in which of the following fluid compartments? a. Intracellular fluid (ICF) b. Extracellular fluid (ECF) c. Interstitial fluid d. Intravascular fluid

d. Intravascular fluid Rationale: Blood plasma is the intravascular fluid. ICF is fluid in the cells. ECF is all the fluid outside the cells. Interstitial fluid is fluid between the cells and outside the blood vessels.

A 42-year-old female presents to her primary care provider reporting muscle weakness and cardiac abnormalities. Laboratory tests indicate that she is hypokalemic. Which of the following could be the cause of her condition? a. Respiratory acidosis b. Constipation c. Hypoglycemia d. Primary hyperaldosteronism

d. Primary hyperaldosteronism Rationale: Primary hyperaldosteronism, with excessive secretion of aldosterone from an adrenal adenoma (tumor) also causes potassium wasting. Acidosis is related to hyperkalemia, not hypokalemia. Constipation can occur with hypokalemia but does not cause it. Hypoglycemia is not related to muscle weakness.

Which patient should the nurse assess for both hyperkalemia and metabolic acidosis? A patient diagnosed with: a. Diabetes insipidus b. Pulmonary disorders c. Cushing syndrome d. Renal failure

d. Renal failure Rationale: Renal failure is associated with hyperkalemia and metabolic acidosis. Diabetes insipidus results in hypernatremia. Pulmonary disorders are a cause of respiratory acidosis or alkalosis but do not affect hyperkalemia. Cushing syndrome results in hypernatremia.

When planning care for a dehydrated patient, the nurse remembers the principle of water balance is closely related to _____ balance. a. Potassium b. Chloride c. Bicarbonate d. Sodium

d. Sodium Rationale: Because water follows the osmotic gradients established by changes in salt concentration, water balance is tied to sodium balance.

When a nurse is teaching about urinary pathogens in men. Which information should the nurse include? Mechanisms for defense against urinary pathogens in men include (select all that apply):

- The long length of the urethra - Antimicrobial secretions from the prostate

A healthcare professional is educating a community men's group about prostate cancer. What information should the professional include in the presentation? (Select all that apply.)

-Being over 65 years of age increases the risk for developing prostate cancer. -A familial history of prostate cancer is a risk factor.

What are the common clinical manifestations of endometriosis?

-Dysmenorrhea -Infertility

A 35-year-old female with Graves disease is admitted to a medical-surgical unit. While the nurse is reviewing the lab tests, which results would the nurse expect to find?

-High levels of circulating thyroid-stimulating antibodies - High levels of circulating thyroid-stimulating antibodies AND elevated T3 AND Intolerance to heat, tachycardia, and weight loss

A 12-year-old female is newly diagnosed with type 1 diabetes mellitus (DM). Which of the following are correct statements? Select ALL that apply.

-It is caused by the immune mediated destruction of the pancreas -Glycosylated hemoglobin measurements, testing for the presence C peptide, fasting plasma glucose levels, and glucose tolerance tests are diagnostic tests for diabetes

A woman has a pelvic organ prolapse. What treatments does the healthcare professional teach the women about? (select all)

-Kegel exercises -estrogen therapy

Elevated levels of glucocorticoids result in which of the following assessment findings? (Select all that apply.)

-Polycythemia -Decreased Calcium -Increased appetite

The nurse is explaining clinical manifestations of alterations in the extrapyramidal system. The nurse would correctly include (select all that apply):

-Presence of tremor -Little or no paralysis of voluntary movement -Rigidity in muscle tone -Normal or slightly increased tendon reflexes

A nurse is preparing to teach a patient about Addison disease. Which information is correct. Select all that apply.

-The most common cause of Addison disease is an autoimmune reaction. -Addison disease is characterized by inadequate corticosteroid and mineralocorticoid synthesis and elevated serum ACTH.

A nurse is assessing a patient with hypoparathyroidism. Clinical manifestations of hypoparathyroidism include (select all that apply)

-Trousseau sign -Hair loss -Tetany -Chvostek sign

A healthcare professional reads in a woman's chart that she has a grade 2 uterine prolapse. What assessment finding does the professional correlate with this condition?

-Uterus reaches the hymen -Uterus protruding from the vagina

The nurse assesses a patient for long term complications of diabetes. This assessment includes evaluations for the following. Select all that apply

-auscultation of carotids for bruits associated with atherosclerosis. -pedal pulse palpation for arterial insufficiency. -auscultation of carotids for bruits associated with atherosclerosis.

A nurse realizes a patient has thrombocytopenia when the platelet count is below _____ platelets per cubic millimeter. 1.

100,000

Physiologic pH is maintained around 7.4 because carbonic acid and bicarbonate exist in a ratio of:

1:20

Physiologic pH is maintained around 7.4 because carbonic acid and bicarbonate exist in a ratio of: 10:2 2:10 1:20 20:1

1:20

A 35-year-old male was diagnosed with hepatitis B. Further tests revealed neutropenia characterized by a neutrophil count less than _____ per milliliter. 1.

2,000

On average the kidneys receive approximately _____ of the cardiac output.

20% to 25%

A nurse is monitoring a patient for pulmonary hypertension. Which mean pulmonary artery pressure reading will indicate to the nurse that pulmonary hypertension is occurring? Pulmonary artery pressure above _____ mm Hg.

25

When the spouse of a patient experiencing a thrombotic stroke asks when "clot busting" treatment should begin, how should the nurse respond? Recommendations suggest that treatment for a thrombotic stroke begin within_______hours of onset of symptoms

3

A 35-year-old male weighs 70 kg. Approximately how much of this weight is ICF?

42 L

Normal intracranial pressure is _____ mm Hg

5 - 15 Intracranial pressure normally is 5 to 15 mm Hg, or 60 to 180 cm H20. p. 557

A criterion for a diagnosis of generalized anxiety disorder (GAD) is a period of excessive worrying that lasts for at least how many months?

6

Which of the following patients is at highest risk for developing pulmonary embolism (PE)?

72-year-old male who is recovering from hip replacement surgery in the hospital

A 20-year-old female suffered from spinal cord injury that resulted from a motor vehicle accident. She had spinal shock lasting 15 days and is now experiencing an uncompensated cardiovascular response to sympathetic stimulation. What does the nurse suspect caused this condition?

A distended bladder or rectum

A 15-year-old female presents with breast discharge, dysmenorrhea, and excessive excitability. Tests reveal that all her pituitary hormones are elevated. What does the nurse suspect as the most likely cause for these assessment findings?

A pituitary adenoma

A 15-year-old female presents with breast discharge, dysmenorrhea, and excessive excitability. Tests reveal that all her pituitary hormones are elevated. What does the nurse suspect as the most likely cause for these assessment findings? A. Phenochromocytoma B. A pituitary adenoma C. Hypothalamic inflammation D. Hypothalamic hyposecretion

A pituitary adenoma

A patient presents with breast discharge, dysmenorrhea, and excessive excitability. Tests reveal that all pituitary hormones are elevated. What does the nurse suspect as the most likely cause for these assessment findings?

A pituitary adenoma

After a partial gastrectomy or pyloroplasty, clinical manifestations that include increased pulse, hypotension, weakness, pallor, sweating, and dizziness are a result of:

A rapid gastric emptying and creation of a high osmotic gradient in the small intestine that causes a sudden shift of fluid from the blood vessels to the intestinal lumen (Early dumping syndrome)

A 50-year-old diabetic male did not take his medication and is now in metabolic acidosis. He is experiencing Kussmaul respirations. What type of breathing will the nurse observe upon assessment?

A slightly increased ventilatory rate, large tidal volumes, and no expiratory pause

A 60-year-old male with a 30-year history of smoking is diagnosed with a hormone-secreting lung tumor. Further testing indicates that the tumor secretes ADH. Which of the following assessment findings should the nurse expect? (Select all that apply.) a. Confusion b. Weakness c. Nausea d. Muscle twitching e. Weight loss

A, B, C, D Rationale: Secretion of ADH leads to water intoxication with symptoms of cerebral edema, with confusion and convulsions; weakness; nausea; muscle twitching; headache; and weight gain, not loss.

The nurse would anticipate the patient with syndrome of inappropriate ADH (SIADH) to demonstrate which of the following symptoms? (Select all that apply.) a. Weakness b. Nausea c. Headache d. Weight loss e. Muscle twitching

A, B, C, E Rationale: Weakness, nausea, muscle twitching, headache, and weight gain, not loss, are common symptoms of chronic water accumulation.

A nurse recalls posthemorrhagic anemia can result in death when a patient's blood loss is in excess of: A. 40% to 49% B. 20% to 29% C. 30% to 39% D. 50% to 59%

A. 40% to 49%

A 15-year-old female presents with breast discharge, dysmenorrhea, and excessive excitability. Tests reveal that all her pituitary hormones are elevated. What does the nurse suspect as the most likely cause for these assessment findings? A. A pituitary adenoma B. Pheochromocytoma C. Hypothalamic inflammation D. Hypothalamic hyposecretion

A. A pituitary adenoma

A nurse checks lab results as both Cushing syndrome and Addison disease can manifest with elevated levels of: A. Adrenocorticotropic hormone (ACTH) B. ADH C. Cortisol D. Aldosterone

A. Adrenocorticotropic hormone (ACTH)

Which statement indicates teaching was successful regarding the classic pathway of the complement system? The classic pathway of the complement system is activated by: A. Antigen-antibody complexes B. Prostaglandins C. Histamine D. Leukotrienes

A. Antigen-antibody complexes

Which of the following is responsible for initiating clonal selection? A. Antigens B. T cells C. B cells D. Lymphocytes

A. Antigens

A 30-year-old male is having difficulty breathing and has been spitting blood. He reports that he began experiencing this reaction after cleaning his pigeons' cages. Testing reveals he is suffering from allergic alveolitis. Which of the following is he experiencing? A. Arthus reaction B. Raynaud phenomenon C. Serum sickness D. Antibody-dependent cytotoxicity

A. Arthus reaction

A common pathway of irreversible cell injury involves increased intracellular: A. Calcium B. Sodium C. Magnesium D. Potassium

A. Calcium

A 5-year-old female is diagnosed with acute leukemia. The nurse will most likely treat this patient with: A. Chemotherapy B. Bone marrow transplant C. Localized radiation therapy D. Immunotherapy

A. Chemotherapy

In disseminated intravascular coagulation (DIC), the nurse assesses for active bleeding after intravascular clotting because: A. Clotting factors are depleted. B. Prothrombin is activated. C. Inflammatory mediators are released. D. Tissue factor (TF) is inactivated.

A. Clotting factors are depleted.

A 30-year-old male was involved in a motor vehicle accident. The glass from the shattered window cut his face and neck. The scar, however, was raised and extended beyond the original boundaries of the wound. This pattern of scarring is caused by impaired: A. Collagen synthesis B. Epithelialization C. Contraction D. Nutritional status

A. Collagen synthesis

A hematologist is discussing hematopoiesis. Which information should be included? ________ participate in hematopoiesis. A. Colony-stimulating factors (CSFs) B. Eosinophils C. Basophils D. Neutrophils

A. Colony-stimulating factors (CSFs)

Confirmation of somatic death is based on: A. Complete cessation of respiration and circulation B. Presence of livor mortis C. Presence of algor mortis D. Change in skin color to pale yellow

A. Complete cessation of respiration and circulation

A 44-year-old patient with pulmonary tuberculosis (lung infection) is evaluated for SIADH. Which of the following assessment findings would be expected in this patient? A. Concentrated urine B. Tachycardia C. Peripheral edema D. Low blood pressure

A. Concentrated urine

A 19-year-old female with type 1 DM was admitted to the hospital with altered consciousness and the following lab values: serum glucose 500 mg/dl (high) and serum K+ 2 (low). Her parents state that she has been sick with the "flu" for a week. The diagnosis is hyperosmolar hyperglycemia nonketotic syndrome (HHNKS). What relationship do these values have with her insulin deficiency? A. Decreased insulin causes hyperglycemia and osmotic diuresis. B. Increased glucose utilization causes the shift of fluid from the intravascular to the intracellular space. C. Increased glucose and fatty acid metabolism stimulates renal diuresis and electrolyte loss. D. Increased insulin use results in protein catabolism, tissue wasting, and electrolyte loss.

A. Decreased insulin causes hyperglycemia and osmotic diuresis.

Which of the following patients is the most at risk for developing hypernatremia? A patient with: A. Dehydration B. Vomiting C. Diuretic use D. Hypoaldosteronism

A. Dehydration

A 30-year-old male presents to his primary care provider reporting visual disturbances. CT reveals a pituitary tumor and lab tests reveal elevated prolactin. He is diagnosed with prolactinoma. Which of the following treatments would the nurse help implement? Administering: A. Dopaminergic agonists B. Calcium C. Radiation D. Insulin

A. Dopaminergic agonists

The nurse is assessing a patient with a diagnosis of inflammation. The nurse would expect to find which of the following signs and symptoms consistent with acute inflammation? (Select all that apply.) A. Erythema B. Pain C. Loss of function D. Exudates E. Heat F. Swelling

A. Erythema B. Pain C. Loss of function E. Heat F. Swelling

A staff member wants to know where the greatest proportion of iron is located. How should the nurse respond? The greatest proportion of total body iron is located in the: A. Erythrocytes B. Spleen pulp C. Bone marrow D. Liver tissue

A. Erythrocytes

A patient has microcytic hypochromic anemia. Which of the following pathogenic mechanisms may cause anemia in this patient? (Select all that apply.) A. Failure of mechanisms of compensatory erythropoiesis B. Increased basal metabolic rate C. Decreased erythrocyte life span D. Swelling in the tissues E. Disturbances of the iron cycle

A. Failure of mechanisms of compensatory erythropoiesis C. Decreased erythrocyte life span E. Disturbances of the iron cycle

Physical barriers that offer the body protection from damage and infection are located in the (select all that apply): A. Genitourinary tract B. Gastrointestinal tract C. Lymph system D. Hematopoietic system E. Respiratory tract

A. Genitourinary tract B. Gastrointestinal tract E. Respiratory tract

An immunologist is discussing endotoxin production. Which information should the immunologist include? Endotoxins are produced by: A. Gram-negative bacteria B. Gram-negative fungi C. Gram-positive fungi D. Gram-positive bacteria

A. Gram-negative bacteria

A 62-year-old female tells her health care provider she has been experiencing regular night sweats that cause her to wake up drenched. She also remarks that she has been unintentionally losing weight. Physical exam reveals enlarged lymph nodes on her neck that do not appear to be painful. She should be screened for which of the following cancers? A. Hodgkin lymphoma B. Epstein-Barr virus C. Acute leukemia D. Burkitt lymphoma

A. Hodgkin lymphoma

A 54-year-old male intravenous (IV) drug user is diagnosed with chronic hepatitis C. Testing revealed that he is a candidate for treatment. Which of the following could be used to treat his condition? A. INFs B. Interleukin-10 C. Interleukin-6 D. Interleukin-1

A. INFs

Which statement indicates a correct understanding of antibodies? The most abundant class of antibody in the serum is: A. IgG B. IgM C. IgE D. IgA

A. IgG

The nurse is reviewing the lab data of a newly admitted patient. The nurse notes the patient had an erythrocyte sedimentation done, and the results are quite elevated. The nurse would focus the care plan on which of the following conditions? A. Inflammation B. Electrolyte imbalance C. Anemia D. Infection

A. Inflammation

During an infection, the nurse assesses the lymph nodes. Lymph nodes enlarge and become tender because: A. Lymphocytes are rapidly dividing. B. Edema accumulates within the fibrous capsule. C. Microorganisms are accumulating. D. The nodes are not functioning properly.

A. Lymphocytes are rapidly dividing.

A nurse recalls that monocytes are blood cells that mature (differentiate) into: A. Macrophages B. Neutrophils C. Eosinophils D. Mast cells

A. Macrophages

A nurse recalls bacteria become resistant to antimicrobials by: A. Mutation B. Attenuation C. Proliferation D. Specialization

A. Mutation

A 50-year-old male patient presents with polyuria and extreme thirst. He was given exogenous ADH. For which of the following conditions would this treatment be effective? A. Neurogenic diabetes insipidus B. Psychogenic diabetes insipidus C. SIADH D. Nephrogenic diabetes insipidus

A. Neurogenic diabetes insipidus

A 20-year-old male shoots his hand with a nail gun while replacing roofing shingles. Which of the following cell types would be the first to aid in killing bacteria to prevent infection in his hand? A. Neutrophils B. Leukotrienes C. Monocytes D. Eosinophils

A. Neutrophils

A 25-year-old female has a heavy menses during which she loses a profuse amount of blood. Which of the following adaptations should the nurse expect? A. Peripheral vasoconstriction B. Movement of fluid into the cell C. Decreased oxygen release from hemoglobin D. Decreased cardiac output

A. Peripheral vasoconstriction

A 40-year-old female is diagnosed with SLE. Which of the following findings would be considered a symptom of this disease? A. Photosensitivity B. Gastrointestinal ulcers C. Decreased glomerular filtration rate D. Rash on trunk and extremities Reset Selection

A. Photosensitivity

A patient has been reading on the Internet that light to moderate intake of alcohol is cardioprotective. When the patient asks the nurse what this means, the nurse should respond that the heart is protected by which of the following mechanisms? (Select all that apply.) A. Prevention of clot formation B. Reduction in platelet aggregation C. Increased collateral circulation D. Vasodilation E. Increased levels of high-density lipoprotein cholesterol F. Increasing fibrinolysis

A. Prevention of clot formation B. Reduction in platelet aggregation E. Increased levels of high-density lipoprotein cholesterol F. Increasing fibrinolysis

Which patient should the nurse assess for both hyperkalemia and metabolic acidosis? A patient diagnosed with: A. Renal failure B. Cushing syndrome C. Diabetes insipidus D. Pulmonary disorders

A. Renal failure

A 54-year-old male with a long history of smoking complains of excessive tiredness, shortness of breath, and overall ill feelings. Lab results reveal decreased pH, increased CO2 , and normal bicarbonate ion. These findings help to confirm the diagnosis of: A. Respiratory acidosis B. Metabolic acidosis C. Respiratory alkalosis D. Metabolic alkalosis

A. Respiratory acidosis

A 40-year-old female develops disseminated intravascular coagulation (DIC). Upon obtaining the history, which finding is the most likely cause of this condition? A. Sepsis B. Blood transfusion C. Immune thrombocytopenic purpura (ITP) D. Snake bite

A. Sepsis

A 20-year-old female undergoes lab testing for anemia. Results show high iron, bilirubin, and transferrin and low hemoglobin and hematocrit. Which of the following is the most likely diagnosis to be documented on the chart? A. Sideroblastic anemia B. Iron deficiency anemia C. Folate deficiency anemia D. Pernicious anemia

A. Sideroblastic anemia

A 5-year-old male presents with low-set ears, a fish-shaped mouth, and involuntary rapid muscular contraction. Laboratory testing reveals decreased calcium levels. Which of the following diagnosis is most likely? A. T lymphocyte deficiency B. B lymphocyte deficiency C. Combined immunologic deficiency D. Complement deficiency

A. T lymphocyte deficiency

A patient has a disease state that results from the secretion of toxins by bacteria. Which medical diagnosis will the nurse see documented on the chart? A. Tetanus B. Hepatitis C. Malaria D. Smallpox

A. Tetanus

What common neurologic disturbances should the nurse assess for in a patient with a pituitary adenoma? A. Visual disturbances B. Coma C. Confused states D. Breathing abnormalities

A. Visual disturbances

A 60-year-old male with a 30-year history of smoking is diagnosed with a hormone-secreting lung tumor. Further testing indicates that the tumor secretes ADH. Which of the following assessment findings should the nurse expect? (Select all that apply.) A. Weakness B. Confusion C. Muscle twitching D. Nausea E. Weight loss

A. Weakness B. Confusion C. Muscle twitching D. Nausea

If a patient's posterior pituitary is removed, which hormone would the nurse expect to decrease PRF ADH ACTH GH

ADH

If a patient's posterior pituitary is removed, which hormone would the nurse expect to decrease?

ADH

During an IgE-mediated hypersensitivity reaction, the degranulation of mast cells is a result of which receptor action? a. Histamine bound to H2 b. Chemotactic factor binding to the receptor c. Epinephrine bound to mast cells d. Acetylcholine bound to mast cells

ANS: A Histamine bound to H2 results in the degranulation of mast cells during an IgE-medicated hypersensitivity reaction. The other options do not cause this reaction.

Which statement is a description of the characteristics of apoptosis? a. Programmed cell death of scattered, single cells b. Characterized by swelling of the nucleus and cytoplasm c. Unpredictable patterns of cell death d. Results in benign malignancies

ANS: A Apoptosis is an active process of cellular self-destruction, also known as programmed cell death, which is implicated in normal and pathologic tissue changes. The remaining options do not accurately describe the characteristics of apoptosis

What is an example of compensatory hyperplasia? a. Hepatic cells increase cell division after part of the liver is excised. b. Skeletal muscle cells atrophy as a result of paralysis. c. The heart muscle enlarges as a result of hypertension. d. The size of the uterus increases during pregnancy.

ANS: A Compensatory hyperplasia is an adaptive mechanism that enables certain organs to regenerate. For example, the removal of part of the liver leads to hyperplasia of the remaining liver cells (hepatocytes) to compensate for the loss. The other options do not accurately describe the term compensatory hyperplasia.

When mismatched blood is administered causing an ABO incompatibility, the erythrocytes are destroyed by: a. Complement-mediated cell lysis c. Phagocytosis in the spleen b. Phagocytosis by macrophages d. Natural killer cells

ANS: A Erythrocytes are destroyed by complement-mediated lysis in individuals with autoimmune hemolytic anemia or as a result of an alloimmune reaction to ABO-mismatched transfused blood cells. The other options are not involved in an ABO incompatibility reaction

It is true that nondividing cells are: a. Found in gastrointestinal lining c. Incapable of synthesizing DNA b. Affected by hyperplasia d. Affected by only hypertrophy

ANS: A Gastrointestinal lining is made up of rapidly dividing cells. Hyperplasia and hypertrophy take place if the cells are capable of synthesizing DNA; however, only hypertrophy occurs in nondividing cells.

After ovulation, the uterine endometrial cells divide under the influence of estrogen. This process is an example of hormonal: a. Hyperplasia c. Hypertrophy b. Dysplasia d. Anaplasia

ANS: A Hormonal hyperplasia chiefly occurs in estrogen-dependent organs, such as the uterus and breast. After ovulation, for example, estrogen stimulates the endometrium to grow and thicken for reception of the fertilized ovum. The other options do not accurately describe the process identified in the question

Immunoglobulin E (IgE) is associated with which type of hypersensitivity reaction? a. I c. III b. II d. IV

ANS: A Hypersensitivity reactions have been divided into four distinct types: type I (IgE-mediated) hypersensitivity reactions, type II (tissue-specific) hypersensitivity reactions, type III (immune complex-mediated) hypersensitivity reactions, and type IV (cell-mediated) hypersensitivity reactions.

What causes the clinical manifestations of confusion, convulsions, cerebral hemorrhage, and coma in hypernatremia? a. High sodium in the blood vessels pulls water out of the brain cells into the blood vessels, causing brain cells to shrink. b. High sodium in the brain cells pulls water out of the blood vessels into the brain cells, causing them to swell. c. High sodium in the blood vessels pulls potassium out of the brain cells, which slows the synapses in the brain. d. High sodium in the blood vessels draws chloride into the brain cells followed by water, causing the brain cells to swell

ANS: A Hypertonic (hyperosmolar) imbalances result in an extracellular fluid concentration greater than 0.9% salt solution (e.g., water loss or solute gain); cells shrink in a hypertonic fluid (see Table 3-7). This shrinking of cells results in the symptoms described in the question. The other options do not accurately describe the cause of these symptoms as they relate to hypernatremia

What is the single most common cause of cellular injury? a. Hypoxic injury c. Infectious injury b. Chemical injury d. Genetic injury

ANS: A Hypoxia, or lack of sufficient oxygen, is the single most common cause of cellular injury (see Figure 2-8). The other options are not a commonly observed as is the correct option

In hyperkalemia, what change occurs to the cells' resting membrane potential? a. Hypopolarization b. Hyperexcitability c. Depolarization d. Repolarization

ANS: A If extracellular potassium concentration increases without a significant change in intracellular potassium, then the resting membrane potential becomes more positive (i.e., changes from -90 to -80 mV) and the cell membrane is hypopolarized (i.e., the inside of the cell becomes less negative or partially depolarized [increase excitability]).

Which arterial pH will initiate the formation of ammonium (NH4) from ammonia (NH3), referred to as academia, in the tubular lumen of the kidney? a. 7.25 c. 7.55 b. 7.35 d. 7.65

ANS: A Pathophysiologic changes in the concentration of hydrogen ion or base in the blood lead to acid-base imbalances. Acidemia is a state in which the pH of arterial blood is less than 7.35. NH3 is produced from glutamine in the epithelial cell and diffuses to the tubular lumen, where it combines with H+ to form NH4

Raynaud phenomenon is classified as a type III hypersensitivity reaction and is due to: a. Immune complexes that are deposited in capillary beds, blocking circulation b. Mast cells that are bound to specific endothelial receptors, causing them to degranulate and creating a localized inflammatory reaction that occludes capillary circulation c. Cytotoxic T cells that attack and destroy the capillaries so that they are unable to perfuse local tissues d. Antibodies that detect the capillaries as foreign protein and destroy them using lysosomal enzymes and toxic oxygen species

ANS: A Raynaud phenomenon is a condition caused by the temperature-dependent deposition of immune complexes in the capillary beds of the peripheral circulation. None of the other options are involved in causing this condition.

Graves disease (hyperthyroidism) is an example of which type II hypersensitivity reaction? a. Modulation b. Antibody-dependent cell-mediated cytotoxicity c. Neutrophil-mediated damage d. Complement-mediated lysis

ANS: A The antibody reacts with the receptors on the target cell surface and modulates the function of the receptor by preventing interactions with their normal ligands, replacing the ligand and inappropriately stimulating the receptor or destroying the receptor. For example, in the hyperthyroidism (excessive thyroid activity) of Graves disease, autoantibody binds to and activates receptors for thyroid-stimulating hormone (TSH) (a pituitary hormone that controls the production of the hormone thyroxine by the thyroid). The other options are not examples of type II hypersensitivity reactions.

Physiologic pH is maintained at approximately 7.4 because bicarbonate (HCO3) and carbonic acid (H2CO3) exist in a ratio of: a. 20:1 c. 10:2 b. 1:20 d. 10:5

ANS: A The relationship between HCO3 and H2CO3 is usually expressed as a ratio. When the pH is 7.4, this ratio is 20:1 (HCO3:H2CO3). The other options do not accurately identify physiologic pH by the correct ratio of HCO3 and H2CO3

The common hay fever allergy is expressed through a reaction that is mediated by which class of immunoglobulins? a. IgE c. IgM b. IgG d. T cells

ANS: A Type I reactions are mediated by antigen-specific IgE and the products of tissue mast cells (see Figure 9-1). The most common allergies (e.g., pollen allergies) are type I reactions. In addition, most type I reactions occur against environmental antigens and are therefore allergic. The other options do not accurately identify the mediation factor related to hay fever.

Causes of hypocalcemia include: (Select all that apply.) a. Repeated blood administration b. Pancreatitis c. Decreased reabsorption of calcium d. Hyperparathyroidism e. Kidney stones

ANS: A, B Blood transfusions are a common cause of hypocalcemia because the citrate solution used in storing whole blood binds with calcium. Pancreatitis causes a release of lipases into soft-tissue spaces; consequently, the free fatty acids that are formed bind calcium, causing a decrease in ionized calcium. The other options are not recognized causes of hypocalcemia.

The electrolyte imbalance hypokalemia exhibits which clinical manifestations? (Select all that apply.) a. Paralytic ileus b. Sinus bradycardia c. Atrioventricular block d. Dry mucous membranes e. Tetany

ANS: A, B, C A variety of dysrhythmias may occur, including sinus bradycardia, atrioventricular block, paroxysmal atrial tachycardia, and paralytic ileus. The other options are not related to hypokalemia.

An imbalance of potassium can produce which dysfunctions? (Select all that apply.) a. Weakness skeletal muscles b. Cardiac dysrhythmias c. Smooth muscle atony d. Visual impairment e. Hearing loss

ANS: A, B, C Symptoms of hyperkalemia vary, but common characteristics are muscle weakness or paralysis and dysrhythmias with changes in the ECG. A wide range of metabolic dysfunctions may result from hypokalemia. Neuromuscular excitability is decreased, causing skeletal muscle weakness, smooth muscle atony, and cardiac dysrhythmias

Which disorders are considered autoimmune? (Select all that apply.) a. Crohn disease d. Systemic lupus erythematosus b. Addison disease e. Noninsulin-dependent diabetes c. Rheumatoid arthritis

ANS: A, B, C, D Crohn disease, Addison disease, rheumatoid arthritis, and systemic lupus erythematosus are all diseases that result from autoimmune pathologic conditions. Insulin-dependent diabetes is also an autoimmune disorder, but noninsulin-dependent diabetes is not.

Exposure to which of the following could result in a type IV hypersensitivity reaction? (Select all that apply.) a. Poison ivy d. Nickel b. Neomycin e. Detergents c. Dairy products

ANS: A, B, D, E Allergens that primarily elicit type IV allergic hypersensitivities include plant resins (e.g., poison ivy, poison oak); metals (e.g., nickel, chromium); acetylates and chemicals in rubber, cosmetics, detergents; and topical antibiotics (e.g., neomycin).

The electrolyte imbalance called hyponatremia exhibits which clinical manifestations? (Select all that apply.) a. Headache b. Seizures c. Paranoia d. Confusion e. Lethargy

ANS: A, B, D, E Behavioral and neurologic changes characteristic of hyponatremia include lethargy, headache, confusion, apprehension, seizures, and coma. Paranoia is not associated with hyponatremia.

During cell injury caused by hypoxia, sodium and water move into the cell because: a. Potassium moves out of the cell, and potassium and sodium are inversely related. b. The pump that transports sodium out of the cell cannot function because of a decrease in ATP levels. c. The osmotic pressure is increased, which pulls additional sodium across the cell membrane. d. Oxygen is not available to bind with sodium to maintain it outside of the cell.

ANS: B A reduction in ATP levels causes the plasma membrane's sodium-potassium (Na+-K+) pump and sodium-calcium exchange to fail, which leads to an intracellular accumulation of sodium and calcium and diffusion of potassium out of the cell. (The Na+-K+ pump is discussed in Chapter 1.)

Type III hypersensitivity reactions are a result of which of the following? a. Antibodies coating mast cells by binding to receptors that signal its degranulation, followed by the discharge of preformed mediators b. Antibodies binding to soluble antigens that were released into body fluids and the immune complexes being deposited in the tissues c. Tc cells or lymphokine-producing Th1 cells directly attacking and destroying cellular targets d. Antibodies binding to the antigen on the cell surface

ANS: B Antigen-antibody (immune) complexes that are formed in the circulation and then deposited later in vessel walls or extravascular tissues (see Figure 9-3) cause most type III hypersensitivity diseases. The other options do not cause this type of reaction.

Which statement is true concerning an atopic individual? a. They tend to produce less IgE. b. They tend to produce more Fc receptors. c. They tend to attract very few mast cells. d. They tend to produce very high levels of IgM

ANS: B Atopic individuals tend to produce higher quantities of IgE and have more Fc receptors for IgE on their mast cells. The other available options are not true.

What type of necrosis is often associated with pulmonary tuberculosis? a. Bacteriologic c. Liquefactive b. Caseous d. Gangrenous

ANS: B Caseous necrosis, which commonly results from tuberculous pulmonary infection, particularly Mycobacterium tuberculosis, is a combination of coagulative and liquefactive necrosis. The other types of necrosis are not observed in pulmonary tuberculosis

It is true that Kussmaul respirations indicate: a. Anxiety is a cause of respiratory acidosis. b. A compensatory measure is needed to correct metabolic acidosis. c. Diabetic ketoacidosis is the cause of the metabolic acidosis. d. More oxygen is necessary to compensate for respiratory acidosis

ANS: B Deep, rapid respirations (Kussmaul respirations) are indicative of respiratory compensation for metabolic acidosis. The other options are not true.

A hypersensitivity reaction that produces an allergic response is called: a. Hemolytic shock c. Necrotizing vasculitis b. Anaphylaxis d. Systemic erythematosu

ANS: B Examples of systemic anaphylaxis are allergic reactions to beestings, peanuts, and fish. The other options are not accurate examples of hypersensitivity.

The mammary glands enlarge during pregnancy primarily as a consequence of hormonal: a. Atrophy c. Anaplasia b. Hyperplasia d. Dysplasia

ANS: B Hormonal hyperplasia occurs chiefly in estrogen-dependent organs, such as the uterus and breast. The remaining options do not adequately describe the consequence of hormones on breast tissue during pregnancy

In hypoxic injury, sodium enters the cell and causes swelling because: a. The cell membrane permeability increases for sodium during periods of hypoxia. b. ATP is insufficient to maintain the pump that keeps sodium out of the cell. c. The lactic acid produced by the hypoxia binds with sodium in the cell. d. Sodium cannot be transported to the cell membrane during hypoxia.

ANS: B In hypoxic injury, movement of fluid and ions into the cell is associated with acute failure of metabolism and a loss of ATP production. Normally, the presence of ATP and ATPase, the active-transport enzyme, maintains the pump that transports sodium ions out of the cell. In metabolic failure caused by hypoxia, reduced ATP and ATPase levels permit sodium to accumulate in the cell, whereas potassium diffuses outward. The other options do not accurately describe the cause of the swelling caused by hypoxia.

What type of necrosis results from ischemia of neurons and glial cells? a. Coagulative c. Caseous b. Liquefactive d. Gangrene

ANS: B Liquefactive necrosis commonly results from ischemic injury to neurons and glial cells in the brain. The other types of necrosis are not related to ischemic injuries in the brain.

Which type of cell adaptation occurs when normal columnar ciliated epithelial cells of the bronchial lining have been replaced by stratified squamous epithelial cells? a. Hyperplasia c. Dysplasia b. Metaplasia d. Anaplasia

ANS: B Metaplasia is the reversible replacement of one mature cell by another, sometimes a less differentiated cell type. The best example of metaplasia is the replacement of normal columnar ciliated epithelial cells of the bronchial (airway) lining by stratified squamous epithelial cells. The other options do not accurately describe the event in the question.

The existence of hyperkalemia is likely to result in which changes to a person's electrocardiogram (ECG)? a. Flattened U waves c. Depressed ST segments b. Peaked T waves d. Peaked P waves

ANS: B Observed ECG changes include peaked T waves, prolonged PR interval, and absent P wave with a widened QRS complex. The other options are not related to hyperkalemia.

Blood transfusion reactions are an example of: a. Autoimmunity c. Homoimmunity b. Alloimmunity d. Hypersensitivity

ANS: B Only alloimmunity (also termed isoimmunity) occurs when the immune system of one individual produces an immunologic reaction against tissues of another individual.

The most common cause of hypermagnesemia is: a. Hepatitis c. Trauma to the hypothalamus b. Renal failure d. Pancreatitis

ANS: B Renal failure usually causes hypermagnesemia, in which magnesium concentration is greater than 2.5 mEq/L. Hypermagnesemia is not a result of the other options

Raynaud phenomenon is an example of which type of hypersensitivity? a. IV c. II b. III d. I

ANS: B The characteristics of serum sickness are observed in only systemic type III autoimmune diseases such as Raynaud phenomenon.

An excessive use of magnesium-containing antacids and aluminum-containing antacids can result in: a. Hypomagnesemia b. Hypophosphatemia c. Hyponatremia d. Hypokalemia

ANS: B The most common causes of hypophosphatemia are intestinal malabsorption and increased renal excretion of phosphate. Inadequate absorption is associated with vitamin D deficiency, the use of magnesium and aluminum-containing antacids (which bind with phosphorus), long-term alcohol abuse, and malabsorption syndromes. The excessive use of such antacids will not result in the other options.

The calcium and phosphate balance is influenced by which three substances? a. Parathyroid hormone, vasopressin, and vitamin D b. Parathyroid hormone, calcitonin, and vitamin D c. Thyroid hormone, vasopressin, and vitamin A d. Thyroid hormone, calcitonin, and vitamin A

ANS: B Three hormones regulate calcium and phosphate balance: parathyroid hormone (PTH), vitamin D, and calcitonin. Vasopressin, thyroid hormone, and vitamin A do not influence calcium and phosphate balance

Which type of antibody is involved in type I hypersensitivity reaction? a. IgA c. IgG b. IgE d. IgM

ANS: B Type I reactions are only mediated by antigen-specific IgE and the products of tissue mast cells (see Figure 9-1).

Which antibody initially indicates a typical primary immune response? a. IgG c. IgA b. IgM d. IgE

ANS: B Typically, IgM is produced first (primary immune response), followed by IgG against the same antigen. The other options are not involved.

What type of necrosis is associated with wet gangrene? a. Coagulative c. Caseous b. Liquefactive d. Gangrene

ANS: B Wet gangrene develops only when neutrophils invade the site, causing liquefactive necrosis.

When the maternal immune system becomes sensitized against antigens expressed by the fetus, what reaction occurs? a. T-cell immunity c. Fetal immunity b. Alloimmunity d. Autoimmunity

ANS: B Alloimmunity occurs when an individual's immune system reacts against antigens on the tissues of other members of the same species. Sensitization against fetal antigens is not the cause of any other available option

Dehydration can cause which result? (Select all that apply.) a. Moist mucous membranes b. Weak pulses c. Tachycardia d. Polyuria e. Weight loss

ANS: B, C, E Significant water deficit is demonstrated by symptoms of dehydration that include headache, thirst, dry skin and mucous membranes, elevated temperature, weight loss, and decreased or concentrated urine (with the exception of diabetes insipidus). Skin turgor may be normal or decreased. Symptoms of hypovolemia include tachycardia, weak pulses, and postural hypotension

The electrolyte imbalance hypercalcemia exhibits which clinical manifestations? (Select all that apply.) a. Diarrhea b. Calcium based kidney stones c. ECG showing narrow T waves d. Lethargy e. Bradycardia

ANS: B, D, E Fatigue, weakness, lethargy, anorexia, nausea, and constipation are common. Behavioral changes may occur. Impaired renal function frequently develops, and kidney stones form as precipitates of calcium salts. A shortened QT segment and depressed widened T waves also may be observed on the ECG, with bradycardia and varying degrees of heart block

When a tuberculin skin test is positive, the hard center and erythema surrounding the induration are a result of which of the following? (Select all that apply.) a. Histamine d. Products of complement b. T lymphocytes e. Macrophages c. Immune complexes

ANS: B, E The reaction site is infiltrated with only T lymphocytes and macrophages, resulting in a clear hard center (induration) and a reddish surrounding area (erythema)

When antibodies are formed against red blood cell antigens of the Rh system, the blood cells are destroyed by: a. Complement-mediated cell lysis b. Phagocytosis by macrophages c. Phagocytosis in the spleen d. Neutrophil granules and toxic oxygen products

ANS: C Antibodies against platelet-specific antigens or against red blood cell antigens of the Rh system coat those cells at low density, resulting in their preferential removal by phagocytosis in the spleen, rather than by complement-mediated lysis. The other options do not cause this reaction.

What characteristic do atopic individuals have that make them genetically predisposed to develop allergies? a. Greater quantities of histamine c. Greater quantities of IgE b. More histamine receptors d. A deficiency in epinephrine

ANS: C Atopic individuals tend to produce higher quantities of IgE and to have more crystalline fragment (Fc) receptors for IgE on their mast cells. The other options do not cause this reaction.

Removal of part of the liver leads to the remaining liver cells undergoing compensatory: a. Atrophy c. Hyperplasia b. Metaplasia d. Dysplasia

ANS: C Compensatory hyperplasia is an adaptive mechanism that enables certain organs to regenerate. For example, the removal of part of the liver leads to hyperplasia of the remaining liver cells (hepatocytes) to compensate for the loss. The other options do not accurately identify the compensatory process described in the question.

In a type III hypersensitivity reaction, the harmful effects after the immune complexes that are deposited in tissues are a result of: a. Cytotoxic T cells c. Complement activation b. Natural killer cells d. Degranulation of mast cells

ANS: C Complement activation, particularly through the generation of chemotactic factors for neutrophils, causes the harmful effects of immune complex deposition. The neutrophils bind to antibody and C3b contained in the complexes and attempt to ingest the immune complexes. Type III hypersensitivity reactions as described are not the result of any of the other option

During an IgE-mediated hypersensitivity reaction, what causes bronchospasm? a. Bronchial edema caused by the chemotactic factor of anaphylaxis b. Bronchial edema caused by binding of the cytotropic antibody c. Smooth muscle contraction caused by histamine bound to H1 receptors d. Smooth muscle contraction caused by histamine bound to H2 receptors

ANS: C During an IgE-mediated hypersensitivity reaction, only smooth muscle contraction caused by histamine bound to H1 receptors results in bronchospasms.

Dysplasia refers to a(n): a. Abnormal increase in the number of a specific cell type b. True adaptive process at the cellular level c. Modification in the shape of a specific cell type d. Lack of oxygen at the cellular level

ANS: C Dysplasia refers only to abnormal changes in the size, shape, and organization of mature cells.

Causes of hyperkalemia include: a. Hyperparathyroidism and malnutrition b. Vomiting and diarrhea c. Renal failure and Addison disease d. Hyperaldosteronism and Cushing disease

ANS: C Hyperkalemia should be investigated when a history of renal disease, massive trauma, insulin deficiency, Addison disease, use of potassium salt substitutes, or metabolic acidosis exists. The other options are not known to be causes of hyperkalemia

Hypersensitivity is best defined as a(an): a. Disturbance in the immunologic tolerance of self-antigens b. Immunologic reaction of one person to the tissue of another person c. Altered immunologic response to an antigen that results in disease d. Undetectable immune response in the presence of antigens

ANS: C Hypersensitivity is an altered immunologic response to an antigen that results in disease or damage to the host. The other options are not accurate definitions of hypersensitivity.

During cell injury caused by hypoxia, an increase in the osmotic pressure occurs within the cell because: a. Plasma proteins enter the cell. b. The adenosine triphosphatase (ATPase)-driven pump is stronger during hypoxia. c. Sodium chloride enters the cell. d. An influx of glucose occurs through the injured cell membranes.

ANS: C In hypoxic injury, movement of fluid and ions into the cell is associated with acute failure of metabolism and a loss of ATP production. Normally, the pump that transports sodium ions out of the cell is maintained by the presence of ATP and ATPase, the active-transport enzyme. In metabolic failure caused by hypoxia, reduced ATP and ATPase levels permit sodium to accumulate in the cell, whereas potassium diffuses outward. The increase of intracellular sodium increases osmotic pressure, which draws more water into the cell. (Transport mechanisms are described in Chapter 1.) The remaining options do not accurately describe the cell injury that results in increased osmotic pressure caused by hypoxia.

During acidosis, the body compensates for the increase in serum hydrogen ions by shifting hydrogen ions into the cell in exchange for which electrolyte? a. Oxygen b. Sodium c. Potassium d. Magnesium

ANS: C In states of acidosis, hydrogen ions shift into the cells in exchange for intracellular fluid potassium; hyperkalemia and acidosis therefore often occur together. This is not true of the other options.

Graves disease is a result of: a. Increased levels of circulating immunoglobulins b. The infiltration of the thyroid with T lymphocytes c. Autoantibodies binding to thyroid-stimulating hormone (TSH)-receptor sites d. Exposure to acetylates in substances such as rubber

ANS: C In the hyperthyroidism (excessive thyroid activity) of Graves disease, autoantibody binds to and activates receptors for TSH (a pituitary hormone that controls the production of the hormone thyroxine by the thyroid). The other options do not accurately describe the cause of Graves disease.

It is true that when insulin is administered: a. The Na+, K+-ATPase pump is turned off. b. Potassium is moved out of muscle cells. c. The liver increases its potassium levels. d. Glucose transport is impaired.

ANS: C Insulin contributes to the regulation of plasma potassium levels by stimulating the Na+, K+-ATPase pump, thereby promoting the movement of potassium simultaneously into the liver and muscle cells with glucose transport after eating. The other options do not accurately describe the effect of insulin administration.

Insulin is used to treat hyperkalemia because it: a. Stimulates sodium to be removed from the cell in exchange for potassium. b. Binds to potassium to remove it through the kidneys. c. Transports potassium from the blood to the cell along with glucose. d. Breaks down the chemical components of potassium, causing it to be no longer effective.

ANS: C Insulin contributes to the regulation of plasma potassium levels by stimulating the Na+, potassium-adenosine triphosphatase (K+-ATPase) pump, thereby promoting the movement of potassium simultaneously into the liver and muscle cells with glucose transport after eating. The intracellular movement of potassium prevents an acute hyperkalemia related to food intake. The other options do not accurately describe how insulin is used to treat hyperkalemia

A patient's blood gases reveal the following findings: pH, 7.3; bicarbonate (HCO3) 27 mEq/L; carbon dioxide (CO2), 58 mm Hg. What is the interpretation of these gases? a. Respiratory alkalosis b. Metabolic acidosis c. Respiratory acidosis d. Metabolic alkalosis

ANS: C The values provided in this question characterize only acute uncompensated respiratory acidosis.

Chvostek and Trousseau signs indicate which electrolyte imbalance? a. Hypokalemia b. Hyperkalemia c. Hypocalcemia d. Hypercalcemia

ANS: C Two clinical signs of hypocalcemia are the Chvostek sign and Trousseau sign. These clinical signs are not indicative of any of the other options.

A type IV hypersensitivity reaction causes which result? a. Antibodies coating mast cells by binding to receptors that signal its degranulation, followed by the discharge of preformed mediators b. Antibodies binding to soluble antigens that were released into body fluids and the immune complexes being deposited in the tissues c. Lymphokine-producing Th1 cells directly attacking and destroying cellular targets d. Antibodies binding to the antigen on the cell surface

ANS: C Types I, II, and III hypersensitivity reactions are mediated by antibody, type IV reactions are mediated by T lymphocytes and do not involve antibody. Type IV mechanisms occur through either Tc cells or lymphokine-producing Th1 cells. Tc cells directly attack and destroy cellular targets.

Vomiting-induced metabolic alkalosis, resulting in the loss of chloride, causes: a. Retained sodium to bind with the chloride b. Hydrogen to move into the cell and exchange with potassium to maintain cation balance c. Retention of bicarbonate to maintain the anion balance d. Hypoventilation to compensate for the metabolic alkalosis

ANS: C When vomiting with the depletion of ECF and chloride (hypochloremic metabolic alkalosis) causes acid loss, renal compensation is not effective; the volume depletion and loss of electrolytes (sodium [Na+], potassium [K+], hydrogen [H+], chlorine [Cl-]) stimulate a paradoxic response by the kidneys. The kidneys increase sodium and bicarbonate reabsorption with the excretion of hydrogen. Bicarbonate is reabsorbed to maintain an anionic balance because the ECF chloride concentration is decreased. The other options do not accurately describe the mechanism that results from vomiting-induced metabolic alkalosis.

Where are antibodies produced? a. Helper T lymphocytes c. Plasma cells b. Thymus gland d. Bone marrow

ANS: C An antibody or immunoglobulin is a serum glycoprotein produced only by plasma cells in response to a challenge by an immunogen.

What organs are affected by the type of necrosis that results from either severe ischemia or chemical injury? (Select all that apply.) a. Lungs b. Brain c. Kidneys d. Muscles e. Heart

ANS: C, E Coagulative necrosis, which occurs primarily in the kidneys, heart, and adrenal glands, is a common result of hypoxia from severe ischemia or hypoxia caused by chemical injury, especially the ingestion of mercuric chloride. The other options do not accurately identify organs affected by necrosis resulting from ischemia or chemical injury.

When soluble antigens from infectious agents enter circulation, tissue damage is a result of: a. Complement-mediated cell lysis b. Phagocytosis by macrophages c. Phagocytosis in the spleen d. Neutrophil granules and toxic oxygen products

ANS: D Of the options available, only the components of neutrophil granules damage the tissue

The abnormal proliferation of cells in response to excessive hormonal stimulation is called: a. Dysplasia c. Hyperplasia b. Pathologic dysplasia d. Pathologic hyperplasia

ANS: D Pathologic hyperplasia is the abnormal proliferation of normal cells and can occur as a response to excessive hormonal stimulation or the effects of growth factors on target cells (see Figure 2-4). The other options do not accurately identify the term for the results of excessive hormonal stimulation on cells.

What is the mechanism that results in type II hypersensitivity reactions? a. Antibodies coat mast cells by binding to receptors that signal its degranulation, followed by a discharge of preformed mediators. b. Antibodies bind to soluble antigens that were released into body fluids, and the immune complexes are then deposited in the tissues. c. Cytotoxic T lymphocytes or lymphokine-producing helper T 1 cells directly attack and destroy cellular targets. d. Antibodies bind to the antigens on the cell surface.

ANS: D The mechanism that results in a type II hypersensitivity reaction begins with antibody binding to tissue-specific antigens or antigens that have attached to particular tissues. The cell can be destroyed by antibody IgG or IgM and activation of the complement cascade through the classical pathwa

How are target cells destroyed in a type II hypersensitivity reaction? a. Complement-mediated cell lysis b. Phagocytosis by macrophages c. Neutrophil granules and toxic oxygen products d. Natural killer cells

ANS: D The mechanism that results in a type II hypersensitivity reaction involves a subpopulation of cytotoxic cells that are not antigen specific (natural killer [NK] cells). Antibody on the target cell is recognized by Fc receptors on the NK cells, which releases toxic substances that destroy the target cell. The other options do not cause the destruction of target cells related to a type II hypersensitivity reaction.

Urticaria are a manifestation of a which type of hypersensitivity reaction? a. IV c. II b. III d. I

ANS: D Urticaria, or hives, is a dermal (skin) manifestation of only type I allergic reactions.

A 26-year-old female recently underwent surgery and is now experiencing dyspnea, cough, fever, and leukocytosis. Tests reveal that she has a collapsed lung caused by removal of air from obstructed alveoli. What condition will the nurse observe on the chart?

Absorption atelectasis

A 19-year-old male presents to his primary care provider reporting restlessness, muscle cramping, and diarrhea. Lab tests reveal that he is hyperkalemic. Which of the following could have caused his condition?

Acidosis

A 19-year-old male presents to his primary care provider reporting restlessness, muscle cramping, and diarrhea. Lab tests reveal that he is hyperkalemic. Which of the following could have caused his condition? Diuretic use Insulin secretion Acidosis Primary hyperaldosteronism

Acidosis

Which of the following conditions would cause the nurse to monitor for hyperkalemia?

Acute acidosis

A 42-year-old female presents with dyspnea; rapid, shallow breathing; inspiratory crackles; decreased lung compliance; and hypoxemia. Tests reveal a fulminant form of respiratory failure characterized by acute lung inflammation and diffuse alveolocapillary injury. Which of the following is the most likely diagnosis the nurse will observe on the chart?

Acute respiratory distress syndrome (ARDS)

Transchondral fractures are most prevalent in:

Adolescents

The nurse is teaching the staff about aldosterone. Which information should the nurse include? The main site of aldosterone synthesis is the:

Adrenal cortex

A nurse checks lab results as both Cushing syndrome and Addison disease can manifest with elevated levels of:

Adrenocorticotropic hormone (ACTH)

A healthcare professional is educating a community men's group about prostate cancer. What information should the professional include in the presentation?

African-American men have an increased risk for its development.

A 60-year-old female with a 25-year history of smoking is diagnosed with emphysema. She has an increased anterior-posterior chest diameter. The nurse attributes this finding to:

Air trapping

A 30-year-old male prison inmate contracted tuberculosis during an outbreak. While planning interactions, the nurse realizes the patient can transmit this disease through:

Airborne droplets

Hepatic fat accumulation is seen in which form of cirrhosis?

Alcoholic

The most common cause of toxic myopathy is:

Alcoholism

The body's inability to conserve water and sodium when affected by Addison disease is explained by which of the following conditions?

Aldosterone deficiency

The body's inability to conserve water and sodium when affected by Addison disease is explained by which of the following conditions? A. Decreased levels of ACTH B. Aldosterone deficiency C. Elevated levels of cortisol D. Hypersecretion of ADH

Aldosterone deficiency The symptoms of Addison disease are primarily a result of hypocortisolism and hypoaldosteronism.

The body's inability to conserve water and sodium and has weakness and fatigability when affected by Addison disease is explained by which of the following conditions?

Aldosterone deficiency and Hypocortisolism

A 53-year-old male with a 20-year history of smoking is diagnosed with emphysema. When the nurse is asked what causes this, what is the nurse's best response? Changes in his lungs are caused by:

Alpha-1-antitrypsin deficiency

Which description is consistent with a focal to bilateral tonic-clonic seizure?

Alternative tonic and clonic movements

When the pulmonologist discusses the condition in which a series of alveoli in the left lower lobe receive adequate ventilation but do not have adequate perfusion, which statement indicates the nurse understands this condition? When this occurs in a patient it is called:

Alveolar dead space

Which of the following shows a correct cause and effect sequence in the development of acute respiratory distress syndrome (ARDS)?

Alveolocapillary membrane injury causing a massive inflammatory response

Which neurologic disorder is characterized by cortical nerve cell processes that have become twisted and dilated?

Alzheimer disease

Researchers now believe that RA is:

An autoimmune disease

A nurse is preparing to teach a patient about Addison disease. What information should the nurse include? The most common cause of Addison disease is:

An autoimmune reaction

What is the most common cause of Addison disease?

An autoimmune reaction

A 60-year-old female with a recent history of head trauma and a long-term history of hypertension presents to the ER for changes in mental status. MRI reveals that she had a hemorrhagic stroke. What does the nurse suspect caused this type of stroke?

Aneurysm

When a patient's renal system secretes rennin, what effect will that cause in the body? It causes the direct activation of:

Angiotensin I

How does SIADH cause excess water?

Ans: The increase in antidiuretic hormone causes retention of water in the renal tubules. Exp: SIADh is not caused by excess water intake but by increased renal absorption of water as a result of inappropriate increases in ADH. Serum sodium and osmolality are reduced by dilution. The kidney continues to excrete sodium, and urine sodium and urine osmolality are elevated; water is reabsorbed, increasing body fluid volume, and urine volume is decreased.

A nurse recalls prolactin-inhibiting factor's target tissue is the:

Anterior Pituitary

A nurse recalls prolactin-inhibiting factor's target tissue is the:

Anterior pituitary

While planning care for a patient with an extradural hematoma, which principle should the nurse remember? The main source of bleeding in extradural (epidural) hematomas is:

Arterial

A 54-year-old female was recently diagnosed with degenerative joint disease. This condition is characterized by loss of:

Articular cartilage

A man reports to the healthcare professional that he had a sudden onset of malaise, low back pain, and perineal pain with high fever and chills, dysuria, nocturia, and urinary retention. What action by the healthcare professional is most appropriate?

Assist the man in obtaining a urine sample.

A patient is admitted to the medical unit for complications of long-term, poorly controlled type 2 DM. Which of the following would the nurse expect to find in addition to elevated glucose?

Atherosclerosis

What pathologic alteration produces tremors at rest, rigidity, akinesia, and postural abnormalities?

Atrophy of neurons in the substantia nigra that produce dopamine

Which would be considered a positive symptom of schizophrenia?

Auditory hallucinations

When a patient has a peculiar sensation that immediately precedes a seizure, what term should the nurse use to describe this sensation?

Aura

Six weeks ago a female patient suffered a T6 spinal cord injury. She then developed a blood pressure of 200/120, a severe headache, blurred vision, and bradycardia. What does the nurse suspect the patient is experiencing?

Autonomic hyperreflexia

A compensatory alteration in the diameter of cerebral blood vessels in response to increased intracranial pressure is called:

Autoregulation

The complement, clotting, and kinin systems share which of the following characteristics? A. Granulocyte production B. Activation of a series of proenzymes C. Phagocytosis initiation D. Activated by interferon

B. Activation of a series of proenzymes

During inflammation, the liver is stimulated to release plasma proteins, collectively known as: A. Opsonins B. Acute phase reactants C. Antibodies D. Phagolysosome

B. Acute phase reactants

A 40-year-old male vegetarian is diagnosed with folate deficiency anemia. He reports that he is an alcoholic. Which of the following factors put him at greatest risk for developing his disease? A. Age B. Being alcoholic C. Being vegetarian D. Gender

B. Being alcoholic

A cell that is produced in the thymus and interacts with MHC class II molecules would have which of the following surface proteins? A. Th1 B. CD4 C. CD8 D. Superantigen

B. CD4

A 67-year-old female is admitted to the emergency department with a diagnosis of polycythemia vera. Upon taking the history, the patient will most likely report: A. A pale skin color B. Chest pain C. Hyperactivity D. Decreased blood pressure

B. Chest pain

A 30-year-old female presents with a gunshot wound to the head. The wound has seared edges and a deep penetration of smoke and gunpowder fragments. This wound would be documented as a(n) _____ wound. A. Indeterminate range entrance B. Contact range entrance C. Intermediate range entrance D. Exit

B. Contact range entrance

Which information indicates a good understanding of bacterial vaccines? Most bacterial vaccines contain: A. Synthetic bacteria B. Dead bacteria C. Active bacteria D. Bacterial toxins

B. Dead bacteria

A 25-year-old female presents to her primary care provider reporting vaginal discharge of a white, viscous, and foul-smelling substance. She reports that she has been taking antibiotics for the past 6 months. Which finding will the nurse most likely see on the microorganism report? A. Clostridium difficile overgrowth B. Decreased Lactobacillus C. Decreased Candida albicans D. Streptococcus overgrowth

B. Decreased Lactobacillus

While planning care for an elderly patient, the nurse remembers that increased age is associated with (select all that apply): A. Increased production of antibodies B. Decreased numbers of circulating immune complexes C. Decreased immune function D. Increased T cell function E. Decreased ability to fight infection

B. Decreased numbers of circulating immune complexes C. Decreased immune function E. Decreased ability to fight infection

A 65-year-old male is diagnosed with multiple myeloma. He reports severe pain. This pain can be attributed to: A. Tissue hypoxia B. Destruction of bone tissue C. Neuropathic infiltrations D. Accumulation of toxic proteins

B. Destruction of bone tissue

A child fell off the swing and scraped the right knee. The injured area becomes painful. What else will the nurse observe upon assessment? A. Pale skin at injured site B. Edema at injured site C. Vasoconstriction at injured site D. Decreased RBC concentration at injured site

B. Edema at injured site

Which patient is most prone to metabolic alkalosis? A patient with: A. Hypoaldosteronism B. Excessive loss of chloride (Cl) C. Retention of metabolic acids D. Hyperventilation

B. Excessive loss of chloride (Cl)

Which portion of the antibody is responsible for the biologic functions of antibodies? A. Variable region B. Fc portion C. Heavy chain D. Epitope

B. Fc portion

Outcomes of laboratory tests include an elevated level of natriuretic peptides. Which organ is the priority assessment? a. Lungs b. Heart c. Liver d. Brain

B. Heart Rationale: Elevated natriuretic peptides indicate problems with the heart or the vasculature.

A 30-year-old male was diagnosed with HIV. Which of the following treatments would be most effective? A. Entrance inhibitors B. Highly active antiretroviral therapy (HAART) C. Protease inhibitors D. Reverse transcriptase inhibitors

B. Highly active antiretroviral therapy (HAART)

After initial compensation, what hemodynamic change should the nurse monitor for in a patient who has a reduction in the number of circulating erythrocytes? A. Altered coagulation B. Hyperdynamic circulatory state C. Decreased cardiac output D. Increased viscosity of blood

B. Hyperdynamic circulatory state

When a staff member asks the nurse what causes the chronic complications of DM such as microvascular and macrovascular disease, how should the nurse respond? These complications are primarily related to: A. Hyperinsulinemia B. Hyperglycemia C. Ketone toxicity D. Pancreatic changes

B. Hyperglycemia

A 45-year-old female with Graves disease underwent surgical removal of her thyroid gland. During the postoperative period, her serum calcium was low. The most probable reason for her low serum calcium is: A. Myxedema secondary to surgery B. Hypoparathyroidism caused by surgical injury to the parathyroid glands C. Hyperparathyroidism secondary to Graves disease D. Hypothyroidism resulting from lack of thyroid replacement

B. Hypoparathyroidism caused by surgical injury to the parathyroid glands

Which information would indicate more teaching is needed regarding hypersensitivity reactions? Type _______ hypersensitivity reactions involve an antibody response. A. I B. IV C. III D. II

B. IV

A 10-year-old male is stung by a bee while playing in the yard. He experiences a severe allergic reaction and has to go to the ER. The nurse providing care realizes this reaction is the result of: A. IgA B. IgE C. Toxoids D. IgM

B. IgE

Which of the following patients is most prone to hypochloremia? A patient with: A. Hypokalemia B. Increased bicarbonate intake C. Hypernatremia D. Hypercalcemia

B. Increased bicarbonate intake

A 68-year-old female is admitted to the emergency department with a diagnosis of polycythemia vera (PV). A nurse realizes the patient's symptoms are mainly the result of: A. A decreased erythrocyte count B. Increased blood viscosity C. Rapid blood flow to the major organs D. Vessel injury

B. Increased blood viscosity

The people from which country have the lowest risk for Hodgkin lymphoma? A. Great Britain B. Japan C. United States D. Denmark

B. Japan

After ingestion of lead, what organ systems should the nurse monitor because they are the most sensitive to the effects of lead? (Select all that apply.) A. Heart B. Kidneys C. Brain D. Hematopoietic E. Lungs F. Liver

B. Kidneys C. Brain D. Hematopoietic

A 25-year-old male is in a car accident and sustains a fracture to his left femur with extensive soft tissue injury. The pain associated with the injury is related to: A. Histamine and serotonin B. Kinins and prostaglandins C. Vasoconstriction D. Immune complex formation

B. Kinins and prostaglandins

A 30-year-old female with Graves disease is admitted to a hospital unit for the surgical removal of her thyroid gland. During the postoperative period, the nurse notes that the patient's serum calcium is low. The nurse should observe the patient for which of the following signs/symptoms? A. Abdominal pain and fever B. Laryngeal spasms and hyperreflexia C. Muscle weakness and constipation D. Anorexia, nausea, and vomiting

B. Laryngeal spasms and hyperreflexia

A 10-year-old male is brought to the emergency room (ER) because he is incoherent and semiconscious. CT scan reveals that he is suffering from cerebral edema. This type of edema is referred to as: A. Generalized edema B. Localized edema C. Pitting edema D. Lymphedema

B. Localized edema

Which statement indicates teaching was successful regarding collectins? Collectins are produced by the: A. Bowel B. Lungs C. Integument D. Kidneys

B. Lungs

When phagocytes begin to stick avidly to capillary walls, which process is occurring? A. Integration B. Margination C. Exudation D. Emigration

B. Margination

A patient has a heart attack that leads to progressive cell injury that causes cell death with severe cell swelling and breakdown of organelles. What term would the nurse use to define this process? A. Apoptosis B. Necrosis C. Adaptation D. Pathologic calcification

B. Necrosis

A 30-year-old male was diagnosed with thyroid carcinoma. The lab tests the nurse would most likely find are _____ T3 and T4 levels. A. High B. Normal C. Low D. Variable

B. Normal

A 49-year-old female is diagnosed with hypercortisolism. Which of the following would the nurse expect? A. Weight loss B. Osteoporosis C. Decreased urination D. Hypoglycemia

B. Osteoporosis

A 20-year-old female has an increase in eosinophils. When the patient wants to know the most likely cause of the eosinophilia. What is the nurse's best response? A. Fungal infections and delayed hypersensitivity B. Parasitic invasion and allergic reactions C. Stress and anxiety reactions D. Viral and bacterial infections

B. Parasitic invasion and allergic reactions

A 58-year-old female presents in the clinic presenting with fatigue, weight loss, and tingling in her fingers. Laboratory findings show low hemoglobin and hematocrit, a high mean corpuscular volume, and normal plasma iron. These assessment findings are consistent with which type of anemia? A. Aplastic anemia B. Pernicious anemia C. Hemolytic anemia D. Iron deficiency anemia

B. Pernicious anemia

After a patient is exposed to a specific antigen, B cells will differentiate into: A. B cytotoxic cells B. Plasma cells C. Bursal cells D. Clonal equivalents

B. Plasma cells

An 11-year-old male is newly diagnosed with type 1 DM. Which classic symptoms should the nurse assess the patient for? A. Weakness, vomiting, hypotension, and mental confusion B. Polydipsia, polyuria, polyphagia, and weight loss C. Vomiting; abdominal pain; sweet, fruity breath; dehydration; and Kussmaul breathing D. Recurrent infections, visual changes, fatigue, and paresthesias

B. Polydipsia, polyuria, polyphagia, and weight loss

A nurse is teaching the staff about platelets. Which information should the nurse include? In addition to playing a role in hemostasis, platelets have the ability to: A. Stimulate bone marrow production of erythrocytes B. Release biochemical mediators of inflammation C. Undergo cell division in response to bleeding D. Activate a humoral response

B. Release biochemical mediators of inflammation

A 22-year-old female has a low level of TSH. What condition does the nurse expect the patient is experiencing? A. Autoimmune hypothyroidism B. Secondary hypothyroidism C. Atypical hypothyroidism D. Primary hypothyroidism

B. Secondary hypothyroidism

A nurse recalls an example of an immune-complex-mediated disease is: A. Rheumatoid arthritis B. Serum sickness C. Contact dermatitis D. Bronchial asthma

B. Serum sickness

A 35-year-old female with Graves disease is admitted to a medical-surgical unit. Which of the following symptoms would the nurse expect to find before treatment? A. Slow heart rate, rash B. Skin hot and moist, rapid heart rate C. Constipation, confusion D. Weight gain, cold intolerance

B. Skin hot and moist, rapid heart rate

When planning care for a dehydrated patient, the nurse remembers the principle of water balance is closely related to _____ balance. A. Chloride B. Sodium C. Bicarbonate D. Potassium

B. Sodium

After erythrocytes have circulated for about 120 days, they are removed by macrophages, which are mainly in the: A. Liver B. Spleen C. Appendix D. Bone marrow

B. Spleen

Which of the following statements indicates more teaching is needed regarding secondary lymph organs? ________is/are a secondary lymph organ. A. The spleen B. The liver C. Adenoids D. Peyer patches

B. The liver

When an aide asks the nurse what is a purpose of the inflammatory process, how should the nurse respond? A. To lyse cell membranes of microorganisms B. To prevent infection of the injured tissue C. To create immunity against subsequent tissue injury D. To provide specific responses toward antigens

B. To prevent infection of the injured tissue

A 30-year-old male prison inmate contracted tuberculosis during an outbreak. When the nurse reviews the lab results, the organism that caused this condition is a:

Bacterium

Parkinson's disease is a degenerative disorder of the:

Basal Ganglia

Blacks over 65 years of age

Blacks over 65 years of age

A 70-year-old female is in the hospital for pelvic fracture. She develops pulmonary thromboembolism. The nurse realizes this embolus is composed of:

Blood clot

A 65-year-old female loses her balance while walking in the woods, causing her to fall and hit her head. She loses consciousness and is in a coma for 5 days. She is diagnosed as having diffuse brain injury. Which of the following would most likely occur in this patient?

Blunted Affect

The patient is experiencing an increase in intracranial pressure. This increase results in:

Brain tissue hypoxia

A 45-year-old male undergoes lung transplantation. He now suffers from airway occlusion secondary to fibrosis. Which diagnosis will the nurse see on the chart?

Bronchiolitis obliterans

Hepatitis _____ in children is primarily associated with blood transfusions.

C

During an IgE-mediated hypersensitivity reaction, which leukocyte is activated? a. Neutrophils c. Eosinophils b. Monocytes d. T lymphocytes

C Of the options provided, only eosinophils are activated during IgE-mediated hypersensitivity reactions.

A 15-year-old male suffers from severe hemorrhage following a motor vehicle accident. He is given a blood transfusion, but shortly afterward the red blood cells are destroyed by agglutination and lysis. Which of the following blood type-transfusion type matches would cause this? A. A-A B. AB-O C. A-AB D. B-O

C. A-AB

Which of the following conditions would cause the nurse to monitor for hyperkalemia? A. Excess aldosterone B. Metabolic alkalosis C. Acute acidosis D. Insulin usage

C. Acute acidosis

A 6-year-old female is diagnosed with a bacterial infection of the respiratory system. Which of the following will most likely try to fight the antigen? A. Self-antigens B. Helper T cells C. Antibodies D. Cytotoxic T cells

C. Antibodies

A 30-year-old female presents with hematuria, menorrhagia, and bleeding gums. She is diagnosed with immune thrombocytic purpura (ITP). A nurse realizes the most likely cause is: A. Allergy-induced platelet lysis B. T cell injury to megakaryocytes C. Antibody destruction of platelets D. An immune response to hypersplenism

C. Antibody destruction of platelets

A 30-year-old female complains of fatigue, arthritis, rash, and changes in urine color. Laboratory testing reveals anemia, lymphopenia, and kidney inflammation. Assuming a diagnosis of SLE, which of the following is also likely to be present? A. Anti-CMV antibodies B. Anti-LE antibodies C. Antinuclear antibodies D. Antiherpes antibodies

C. Antinuclear antibodies

The post-surgical patient is experiencing delayed wound healing. The dietician believes the delay is related to nutritional intake. A deficiency in which of the following substances could directly affect healing? A. Cholesterol B. Melanin C. Ascorbic acid D. Vitamin D

C. Ascorbic acid

A 35-year-old male with hyperthyroidism begins treatment to decrease thyroid activity. A nurse monitors for which of the following conditions that could result secondary to the treatment? A. Monocytosis B. Lymphocytosis C. Basophilia D. Eosinophilia

C. Basophilia

An aide asks the nurse what is the most common cause of elevated levels of antidiuretic hormone (ADH) secretion. How should the nurse respond? A. Autoimmune disease B. Pregnancy C. Cancer D. Heart failure

C. Cancer

A nurse remembers the primary actions of the complement cascade include (select all that apply): A. Increased clotting B. Vasoconstriction C. Cell killing D. Chemotaxis E. Increased vascular permeability F. Opsonization

C. Cell killing D. Chemotaxis E. Increased vascular permeability F. Opsonization

A 5-year-old female takes a hike through the woods during a school field trip. Upon returning home, she hugs her father, and he later develops poison ivy. Which of the following immune reactions is he experiencing? A. Tissue specific B. IgE-mediated C. Cell-mediated D. Immune complex

C. Cell-mediated

The directional migration of leukocytes along a chemical gradient is termed: A. Endocytosis B. Diapedesis C. Chemotaxis D. Margination

C. Chemotaxis

A 2-year-old swallowed watch batteries. Following ingestion, kidney function was impaired, and the heart began to fail. Which of the following was the most likely cause? A. Karyorrhexis B. Caseous necrosis C. Coagulative necrosis D. Ammonia accumulation

C. Coagulative necrosis

An experiment is carried out to characterize newly discovered immunoglobulins. Specificity of the amino acids within the immunoglobulin could be determined by studying the: A. C region of the H chain B. C region of the light chain C. Complementary-determining region (CDR) D. Framework regions

C. Complementary-determining region (CDR)

A 22-year-old male is admitted to the intensive care unit with a closed head injury sustained in a motorcycle accident. The injury has caused severe damage to the posterior pituitary. Which of the following complications should the nurse anticipate? A. Dilutional hyponatremia B. Metabolic acidosis C. Dehydration from polyuria D. Cardiac arrest from hyperkalemia

C. Dehydration from polyuria

Biochemical secretions that trap and kill microorganisms include: A. Hormones B. Gastric acid C. Earwax D. Neurotransmitters

C. Earwax

While reviewing lab results, the nurse recalls the most abundant cells in the blood are: A. Leukocytes B. Lymphocytes C. Erythrocytes D. Thrombocytes

C. Erythrocytes

A 22-year-old female just delivered a healthy baby girl. She suffered from eclampsia during her pregnancy, and on the second postpartum day she complained of bleeding gums and bruising on her arms and legs. Hematology lab tests indicate that she had disseminated intravascular coagulation (DIC). Further review of lab tests by the nurse revealed an increase in: A. Protein C B. Platelets C. Fibrin degradation products (FDPs) D. Hematocrit

C. Fibrin degradation products (FDPs)

Self-antigens do not meet the requirements for immunogenicity because they lack which of the following? A. Virulence B. Quantity C. Foreignness D. Size

C. Foreignness

An 86-year-old female patient has the wasting syndrome of aging, making her vulnerable to falls, functional decline, disease, and death. The nurse knows this patient is experiencing: A. Sarcopenia B. Somatic death C. Frailty D. Cellular aging

C. Frailty

The nurse would correctly respond that the etiology of a congenital immune deficiency is due to a(n): A. Adverse response to a medication B. Renal failure C. Genetic defect D. Negative response to an immunization

C. Genetic defect

Which of the following is a characteristic of the human immunodeficiency virus (HIV), which causes AIDS? A. HIV only infects B cells. B. Infection does not require a host cell receptor. C. HIV is a retrovirus. D. After infection, cell death is immediate.

C. HIV is a retrovirus.

Which patient will develop active immunity? A patient who: A. Has T cells that become B cells B. Receives preformed antibodies or T cells from a donor C. Has natural exposure to an antigen or receives an immunization D. Receives immunoglobulin

C. Has natural exposure to an antigen or receives an immunization

Outcomes of laboratory tests include an elevated level of natriuretic peptides. Which organ is the priority assessment? A. Lungs B. Liver C. Heart D. Brain

C. Heart

While checking the lab results for a patient with Graves disease, the nurse would check the T3 level to be abnormally: A. Low B. Absent C. High D. Variable

C. High

A 35-year-old female is diagnosed with vitamin B12 deficiency anemia (pernicious anemia). How should the nurse respond when the patient asks what causes pernicious anemia? A decrease in ______ is the most likely cause. A. Gastric enzymes B. Erythropoietin C. Intrinsic factor D. Ferritin

C. Intrinsic factor

A 21-year-old woman was recently diagnosed with iron deficiency anemia. Her hematocrit is 32%. Which of the following treatments would the nurse expect to be prescribed for her? A. No treatment is necessary B. A bone marrow transplant C. Iron replacement D. Splenectomy

C. Iron replacement

For a patient with respiratory acidosis, chronic compensation by the body will include: A. Protein buffering B. Kidney excretion of HCO3 C. Kidney excretion of H+ D. Prolonged exhalations to blow off CO2

C. Kidney excretion of H+

A nurse is preparing to teach about erythropoietin. Which information should the nurse include? Erythropoietin is produced in the: A. Liver B. Bone marrow C. Kidneys D. Spleen

C. Kidneys

A 15-year-old female presents to the ER following a physical assault. She has internal damage to the neck with deep bruising. X-ray reveals fractures of the hyoid bone and tracheal and cricoid cartilage. Which of the following most likely caused her injuries? A. Ligature strangulation B. Chemical asphyxiation C. Manual strangulation D. Choking asphyxiation

C. Manual strangulation

An unstable type of hemoglobin that cannot bind with oxygen is termed: A. Deoxyhemoglobin B. Oxyhemoglobin C. Methemoglobin D. Glycosylated hemoglobin

C. Methemoglobin

What problem should the nurse assess for in a patient with chronic hyperparathyroidism? A. Hyponatremia B. Vitamin D malabsorption C. Osteoporosis and pathologic fractures D. Seizure disorders

C. Osteoporosis and pathologic fractures

A newborn is diagnosed with congenital intrinsic factor deficiency. Which of the following types of anemia will the nurse see documented on the chart? A. Sideroblastic anemia B. Hemolytic anemia C. Pernicious anemia D. Iron deficiency anemia

C. Pernicious anemia

A nurse is discussing fibrinolysis. Which information should the nurse share? Fibrinolysis is mediated by: A. Heparin B. Fibrinogen C. Plasmin D. Albumin

C. Plasmin

In addition to matching ABO antigens, a blood transfusion must also be matched for: A. Immunoglobulins B. HLA type C. Rh antigen D. Platelet compatibility

C. Rh antigen

A report comes back indicating that muscular atrophy has occurred. A nurse recalls that muscular atrophy involves a decrease in muscle cell: A. Lipofuscin B. Vacuoles C. Size D. Number

C. Size

A patient with end-stage renal disease has pruritus. When the patient asks what causes this, what is the nurse's best response? Pruritus, seen in patients with end-stage renal disease, is caused by high levels of:

Calcium

An aide asks the nurse what is the most common cause of elevated levels of antidiuretic hormone (ADH) secretion. How should the nurse respond?

Cancer

What is the most common cause of elevated levels of antidiuretic hormone (ADH) secretion?

Cancer

Which of the following buffer pairs is considered the major plasma buffering system?

Carbonic acid/bicarbonate

Which condition is considered the ultimate cause of death in the patient with diabetes?

Cardiovascular disease

Which of the following diseases should the nurse teach the patient to prevent as it is the ultimate cause of death in the patient with diabetes? A. Renal disease B. Stroke C. Cancer D. Cardiovascular disease

Cardiovascular disease

When a patient asks about target cell receptors, which is the nurse's best response? Target cell receptors for most water-soluble hormones are located in the:

Cell membrane

A 20-year-old male is brought to the emergency room (ER) for treatment of injuries received in a motor vehicle accident. A spinal cord injury is suspected. What two regions should the nurse assess as they are most likely to be damaged?

Cervical and lumbar regions

Which principle should the nurse remember while planning care for a patient with spinal shock? Spinal shock is characterized by:

Cessation of spinal cord function below the lesion

A 50-year-old male is diagnosed with pulmonary embolism (PE). Which of the following symptoms most likely occurred before treatment?

Chest pain and shortness of breath

A 30-year-old female received a severe head injury in a motor vehicle accident. She is now experiencing respiratory abnormalities characterized by alternating periods of deep and shallow breathing with periods of apnea. What term should the nurse use when charting this condition?

Cheyne-Stokes

A 69-year-old male with a history of alcohol abuse presents to the emergency room (ER) after a month-long episode of headaches and confusion. Based on his alcoholism, a likely cause of his neurologic symptoms is:

Chronic Subdural Hematoma

A 25-year-old male presents with chronic bronchitis of 5 months' duration. When obtaining the patient's history, which of the following findings is most likely to cause this condition?

Cigarette smoke

A 50-year-old male with a 30-year history of smoking was diagnosed with lung cancer. He was previously exposed to air pollution, asbestos, and radiation at his job. Which of the following should the nurse realize had the greatest impact on the development of his cancer?

Cigarette smoke

Which statement is FALSE concerning the accumulation of fluid in the peritoneal cavity?

Circulating nitric oxide causes vasoconstriction, which forces fluid from the capillaries into the peritoneal cavity

Which statement is false concerning the accumulation of fluid in the peritoneal cavity?

Circulating nitric oxide causes vasoconstriction, which forces fluid from the capillaries into the peritoneal cavity.

A nurse recalls asthma is classified by:

Clinical severity

A 27-year-old male has a severe kidney obstruction leading to removal of the affected kidney. Which of the following would the nurse expect to occur?

Compensatory hypertrophy of the remaining kidney

A 44-year-old patient with pulmonary tuberculosis (lung infection) is evaluated for SIADH. Which of the following assessment findings would be expected in this patient?

Concentrated urine

A 44-year-old patient with pulmonary tuberculosis is evaluated for SIADH. Which assessment finding would support this diagnosis?

Concentrated urine

A 44-year-old patient with pulmonary tuberculosis (lung infection) is evaluated for SIADH. Which of the following assessment findings would be expected in this patient? A. Peripheral edema B. Concentrated urine C. Tachycardia D. Low blood pressure

Concentrated urine Clinical manifestations of SIADH include urine that is inappropriately concentrated with respect to serum osmolarity.

A nurse is preparing to teach staff about the most common type of traumatic brain injury. Which type of traumatic brain injury should the nurse discuss

Concussion

A 60-year-old male with a 30-year history of smoking is diagnosed with a hormone-secreting lung tumor. Further testing indicates that the tumor secretes ADH. Which of the following assessment findings should the nurse expect?

Confusion, weakness, nausea, and muscle twitching

A nurse recalls stress-induced stimulation of the adrenal cortex causes it to secrete:

Cortisol

A nurse recalls stress-induced stimulation of the adrenal cortex causes it to secrete: Adrenocorticotropin hormone (ACTH) Estrogen Parathyroid hormone Cortsol

Cortsol

An important and useful clinical measure in the diagnosis of rhabdomyolysis is measurement of which laboratory value?

Creatine kinase (CK)

A 65-year-old female with emphysema presents to the ER for difficulty breathing. Physical exam reveals bluish skin and mucous membranes. How should the nurse chart this condition? Patient has:

Cyanosis

A 67-year-old male was diagnosed with polycythemia vera (PV) but refused treatment. His condition is at risk for converting to: A. Chronic lymphocytic leukemia B. Multiple myeloma C. Burkitt lymphoma D. Acute myeloid leukemia

D. Acute myeloid leukemia

A 5-year-old male is diagnosed with a bacterial infection. Cultures of the bacteria revealed lipopolysaccharides on the bacterial cell surface. Which of the complement pathways would be activated in this case? A. Classical pathway B. Kinin pathway C. Lectin pathway D. Alternative pathway

D. Alternative pathway

A patient has researched white blood cells on the Internet. Which statement indicates the patient has a good understanding? _______________ contain preformed granules of vasoactive amines. A. Neutrophils B. Eosinophils C. Monocytes D. Basophils

D. Basophils

An immunology nurse is caring for a patient. While planning care, which principle will the nurse remember? The primary role of IgA1 is to prevent infections in the: A. Lungs B. Mucous membranes C. Kidneys D. Blood

D. Blood

The nurse will check which of the following tests to directly measure iron stores? A. Serum ferritin B. Total iron-binding capacity C. Transferrin saturation D. Bone marrow biopsy

D. Bone marrow biopsy

A family presents to their primary care provider reporting headache, nausea, weakness, and vomiting. Which of the following would be the most likely explanation for these symptoms? A. Lead exposure B. Mercury poisoning C. Ethanol exposure D. Carbon monoxide poisoning

D. Carbon monoxide poisoning

When the immunologist says that pathogens possess virulence, what does virulence mean? A. Induces an immune response B. Damages tissue C. Spreads from one individual to others and causes disease D. Causes disease

D. Causes disease

Thrombocytopenia may be: A. Transient or consistent B. Normal or abnormal C. Active or inactive D. Congenital or acquired

D. Congenital or acquired

An experiment is designed to determine specific cell types involved in cell-mediated immune response. The experimenter is interested in finding cells that attack cells that have specific antigens. Which cells should be isolated? A. Helper T cells B. Lymphokine-producing cells C. Macrophages D. Cytotoxic T cells

D. Cytotoxic T cells

While planning care for elderly individuals, the nurse remembers the elderly are at a higher risk for developing dehydration because they have a(n): A. Increased tendency towards developing edema B. Higher total body water volume C. Increase in thirst D. Decreased muscle mass

D. Decreased muscle mass

A 52-year-old diabetic male presents to the ER with lethargy, confusion, and depressed reflexes. His wife indicates that he does not follow the prescribed diet and takes his medication sporadically. Lab results indicate hyperglycemia. Which assessment finding is most likely to occur? A. Decreased urine formation B. Clammy skin C. Metabolic alkalosis D. Decreased sodium

D. Decreased sodium

A nurse is caring for a patient who cannot clot. Which end product of the clotting cascade is this patient unable to make? A. Collagen B. Fibrinogen C. Thrombin D. Fibrin

D. Fibrin

A 50-year-old female experiences decreased blood pressure, decreased oxygen delivery, cardiovascular shock, and subsequent death. A complication of endotoxic shock is suspected. Which of the following is the most likely cause? A. Virus B. Fungi C. Gram-positive bacteria D. Gram-negative bacteria

D. Gram-negative bacteria

A 5-year-old male was diagnosed with normocytic-normochromic anemia. Which of the following anemias does the nurse suspect the patient has? A. Sideroblastic anemia B. Iron deficiency anemia C. Pernicious anemia D. Hemolytic anemia

D. Hemolytic anemia

When a patient asks what causes hyperglycemia in type 2 DM, how should the nurse respond? Hyperglycemia is a result of: A. Liver dysfunction B. Glucagon deficiency C. Insulin deficiency D. Hyperinsulinemia

D. Hyperinsulinemia

A 50-year-old female presents with lightheadedness and overall abnormal feelings. Hyperaldosteronism is diagnosed. Which of the following symptoms would the nurse expect? A. Hypovolemia B. Hyponatremia C. Hypotension D. Hypokalemia

D. Hypokalemia

A 30-year-old female presents to her primary care provider reporting fatigue, excessive sweating, and increased appetite. Physical examination reveals protruding eyes, and laboratory testing reveals hyperthyroidism secondary to autoantibody production. This disorder falls into the category of type _____ hypersensitivity. A. IV B. I C. III D. II

D. II

A 70-year-old male with chronic renal failure presents with edema. Which of the following is the most likely cause of this condition? A. Increased capillary oncotic pressure B. Increased interstitial hydrostatic pressure C. Decreased interstitial oncotic pressure D. Increased capillary hydrostatic pressure

D. Increased capillary hydrostatic pressure

A 19-year-old female with type 1 DM was admitted to the hospital with the following lab values: serum glucose 500 mg/dl (high), urine glucose and ketones 4+ (high), and arterial pH 7.20 (low). Her parents state that she has been sick with the "flu" for a week. Which of the following statements best explains her acidotic state? A. Low serum insulin promotes lipid storage and a corresponding release of ketones. B. Her uncontrolled diabetes has led to renal failure. C. Increased insulin levels promote protein breakdown and ketone formation. D. Insulin deficiency promotes lipid metabolism and ketone formation.

D. Insulin deficiency promotes lipid metabolism and ketone formation.

When the immunologist says that pathogens possess infectivity, what is the immunologist explaining? Infectivity allows pathogens to: A. Spread from one individual to others and cause disease B. Induce an immune response C. Damage tissue D. Invade and multiply in the host

D. Invade and multiply in the host

Liquefactive necrosis occurs in the brain because: A. Of protein denaturation B. Ischemia results in chemical injury C. Debris is not digested by hydrolases D. It is rich in hydrolytic enzymes and lipids

D. It is rich in hydrolytic enzymes and lipids

A 52-year-old male IV drug user was diagnosed with hepatitis C 5 years ago. He is now experiencing impaired blood clotting. The nurse suspects a decrease in which of the following vitamins? A. E B. B12 C. D D. K

D. K

While planning care for a patient with inflammation, which principle will the nurse remember? The predominant phagocyte of early inflammation is the: A. Eosinophil B. Lymphocyte C. Macrophage D. Neutrophil

D. Neutrophil

Visual disturbances are a common occurrence in patients with untreated Graves disease. The endocrinologist explains to the patient that the main cause of these complications is: A. Local lactic acidosis B. Decreased blood flow to the eye C. TSH neurotoxicity to retinal cells D. Orbital edema and protrusion of the eyeball

D. Orbital edema and protrusion of the eyeball

When a patient wants to know what most commonly causes hypoparathyroidism, how should the nurse reply? It is most commonly caused by: A. Hypothalamic inactivity B. Pituitary hyposecretion C. Parathyroid adenoma D. Parathyroid gland injury

D. Parathyroid gland injury

A 50-year-old male intravenous drug user is diagnosed with hepatitis C. Examination of the liver reveals cell death secondary to: A. Pyknosis B. Physiologic apoptosis C. Fat necrosis D. Pathologic apoptosis

D. Pathologic apoptosis

A 24-year-old female presents with excessive menstrual bleeding. The physician identified endometrial changes that are due to hormonal imbalances. These cellular changes would be referred to as: A. Hyperplasia B. Dysplasia C. Pathologic dysplasia D. Pathologic hyperplasia

D. Pathologic hyperplasia

When a nurse observes muscle stiffening occurring within 6 to 14 hours after death, the nurse should document this finding as _____ present. A. Algor mortis B. Gangrene C. Livor mortis D. Rigor mortis

D. Rigor mortis

While planning care for a newborn, the pediatric nurse recalls the main site of hematopoiesis in the fetus is the: A. Bone marrow B. Liver C. Lymph nodes D. Spleen

D. Spleen

A 21-year-old female was recently diagnosed with iron deficiency anemia. In addition to fatigue and weakness, which of the following clinical signs and symptoms would she most likely exhibit? A. Gait problems B. Petechiae C. Hyperactivity D. Spoon-shaped nails

D. Spoon-shaped nails

A 50-year-old male sustained a closed head injury as a result of a motor vehicle accident. CT scan revealed a collection of blood between the inner surface of the dura mater and the surface of the brain. Which type of injury will the nurse be caring for? A. Epidural hematoma B. Contusion C. Abrasion D. Subdural hematoma

D. Subdural hematoma

An infant is experiencing hemolytic disease of the newborn. Which of the following would the nurse expect to find in the infant's history and physical? A. The mother was exposed to measles. B. The father was exposed to Agent Orange. C. The baby was born 6 weeks prematurely. D. The baby is Rh positive.

D. The baby is Rh positive.

A student asks the instructor about recycled iron. What information should the instructor share? Recycled iron from macrophages is delivered to the bone marrow bound to: A. Ferritin B. Hemoglobin C. Hemosiderin D. Transferrin

D. Transferrin

A 35-year-old female took corticosteroid therapy for several months. Which of the following would the nurse expect to find? A. Hypotension B. Episodes of hypoglycemia C. Renal toxicity D. Type 2 DM

D. Type 2 DM

Which of the following clinical findings would be expected in the patient with rhabdomyolysis?

Dark urine

When taking care of a patient with hyperkalemia, which principle is priority? Hyperkalemia causes a(n) _____ in resting membrane potential with _____ excitability of cardiac muscle. Increase; decreased Decrease; increased Increase; increased Decrease; decreased

Decrease; increased

A 45-year-old female has elevated thyroxine production. Which of the following would accompany this condition Increased thyroid-releasing hormone (TRH) Increased anterior pituitary stimulation Decreased T4 Decreased thyroid-stimulating hormone (TSH)

Decreased TSH

A 25-year-old male is diagnosed with a hormone-secreting tumor of the adrenal cortex. Which finding would the nurse expect to see in the lab results?

Decreased blood K+ levels

While planning care for a patient with urinary problems, the nurse recalls that the renin-angiotensin system will be activated by:

Decreased blood pressure in the afferent arterioles

A 70-year-old female presents with a hip fracture. She is diagnosed with osteoporosis. One factor that most likely contributed to her condition is:

Decreased estrogen levels

A 15-year-old female is diagnosed with restrictive lung disease caused by fibrosis. The patient had a pulmonary functions test. Which of the following findings is expected?

Decreased functional residual capacity

A group of mountain climbers experience confusion, tachycardia, edema, and decreased renal output after climbing Mount Rainier. A nurse recalls this condition is caused by:

Decreased inspired oxygen

A 14 year old boy, who was previously diagnosed with Type I diabetes, is admitted with the following lab values: arterial pH 7.2; serum glucose 350 mg/dL; urine glucose and ketones 4+/strong. His parents state that he has been sick with the "flu" for a week. What relationship do these values have to his insulin deficiency?

Decreased insulin causes fatty acid use, ketone formation, metabolic acidosis, and solute diuresis

A 19-year-old female with type 1 DM was admitted to the hospital with altered consciousness and the following lab values: serum glucose 500 mg/dl (high) and serum K+ 2 (low). Her parents state that she has been sick with the "flu" for a week. The diagnosis is diabetic ketoacidosis (DKA). What relationship do these values have with her insulin deficiency?

Decreased insulin causes hyperglycemia and osmotic diuresis.

While planning care for elderly individuals, the nurse remembers the elderly are at a higher risk for developing dehydration because they have a(n):

Decreased muscle mass

A 52-year-old diabetic male presents to the ER with lethargy, confusion, and depressed reflexes. His wife indicates that he does not follow the prescribed diet and takes his medication sporadically. Lab results indicate hyperglycemia. Which assessment finding is most likely to occur?

Decreased sodium

A 45-year-old female has elevated thyroxine production. What would accompany this condition?

Decreased thyroid-stimulation hormone

An adult has hydrocephalus. When the patient asks the nurse what caused this, how should the nurse respond? Hydrocephalus in adults is most often caused by:

Defective CSF reabsorption

Which of the following patients is the most at risk for developing hypernatremia? A patient with:

Dehydration

A 22-year-old male is admitted to the intensive care unit with a closed head injury sustained in a motorcycle accident. The injury has caused severe damage to the posterior pituitary. Which of the following complications should the nurse anticipate?

Dehydration from polyuria

A 22-year-old male is admitted to the intensive care unit with a closed head injury sustained in a motorcycle accident. The injury has caused severe damage to the posterior pituitary. Which of the following complications should the nurse anticipate? A. Cardiac arrest from hyperkalemia B. Dilutional hyponatremia C. Metabolic acidosis D. Dehydration from polyuria

Dehydration from polyuria

A patient is admitted to the intensive care unit with a closed head injury sustained in a motorcycle accident. The injury has caused severe damage to the posterior pituitary. Which of the following complications should the nurse anticipate?

Dehydration from polyuria

A 7-year old child has cryptorchidism. What action by the healthcare professional is most appropriate?

Describe the correct administration of GnRH.

A 35-year-old male received a traumatic brain injury in a motor vehicle accident. CT scan revealed a lesion above the pontine micturition center. Which of the following would the nurse expect?

Detrusor hyperreflexia

A 30-year-old white male recently suffered a cerebrovascular accident. Which of the following is the most likely factor that contributed to his stroke?

Diabetes

A 33-year-old male is brought to the ER for treatment of injuries received in a motor vehicle accident. An MRI reveals an injury of the cervical cord. Cord swelling in this region may be life threatening because:

Diaphragm function may be impaired.

Fibromyalgia is a chronic musculoskeletal disorder characterized by:

Diffuse pain, fatigue, and tender points

A 25-year-old female with Graves disease is admitted to a medical-surgical unit. Palpation of her neck would most likely reveal:

Diffuse thyroid enlargement

A 25-year-old female with Graves disease is admitted to a medical-surgical unit. Palpation of her neck would most likely reveal: A. A small discrete thyroid nodule B. A normal-sized thyroid C. Multiple discrete thyroid nodules D. Diffuse thyroid enlargement

Diffuse thyroid enlargement

A patient has researched lipid-soluble hormones on the Internet. Which information indicates the patient has a good understanding Lipid-soluble hormone receptors cross the plasma membrane by: Diffusion Osmosis Active transport Endocytosis

Diffusion

A patient has researched lipid-soluble hormones on the Internet. Which information indicates the patient has a good understanding? Lipid-soluble hormone receptors cross the plasma membrane by:

Diffusion

When insulin binds to its receptors on muscle cells, an increase in glucose uptake by the muscle cells occurs. This is an example of a _____ effect by a hormone.

Direct

When insulin binds to its receptors on muscle cells, an increase in glucose uptake by the muscle cells occurs. This is an example of a ________effect by a hormone.

Direct

A 32-year-old male was injured in a motor vehicle accident and confined to bed for 3 weeks. During this time, the size and strength of muscle fibers decreased, a condition referred to as:

Disuse atrophy

A 30-year-old male presents to his primary care provider reporting visual disturbances. CT reveals a pituitary tumor and lab tests reveal elevated prolactin. He is diagnosed with prolactinoma. Which of the following treatments would the nurse help implement? Administering:

Dopaminergic agonists

A 30-year-old male presents to his primary care provider reporting visual disturbances. CT reveals a pituitary tumor and lab tests reveal elevated prolactin. He is diagnosed with prolactinoma. Which of the following treatments would the nurse help implement? Administering: A. Calcium B. Dopaminergic agonists C. Insulin D. Radiation

Dopaminergic agonists

A patient presents reporting visual disturbances. When CT reveals a pituitary tumor and lab tests reveal elevated prolactin, the diagnosis of prolactinoma is made. Which intervention is the treatment of choice for this condition?

Dopaminergic agonists

A patient has high levels of hormones. To adapt to the high hormone concentrations, the patient's target cells have the capacity for:

Down regulation

Which of the following is consistent with dumping syndrome?

Dumping syndrome usually responds well to dietary management.

When the nurse observes a diagnosis of nosocomial pneumonia, the patient generally acquires this pneumonia:

During hospitalization

A 10-year-old female develops pneumonia. Physical exam reveals subcostal and intercostal retractions. She reports that breathing is difficult and she feels she cannot get enough air. What term should the nurse use to document this condition?

Dyspnea

Individuals with a recent diagnosis of emphysema should be assessed for which most common presenting factor?

Dyspnea

A 30-year-old male is involved in a motor vehicle accident and sustains trauma to the lungs and chest wall. He experiences respiratory failure. Which of the following lab values would the nurse expect?

Elevated PaCO2

What is the cause of the hyperpigmentation seen in people with Cushing syndrome?

Elevated levels of ACTH

A 60-year-old male is diagnosed with renal failure. While the nurse is reviewing lab results, which of the following lab values would be most consistent with this diagnosis?

Elevated plasma creatinine level

An 80-year-old female develops pneumonia in the hospital. She becomes cyanotic, tachycardic, and develops a fever and cough. Chest x-ray reveals pus in the pleural space. Which of the following is the most likely diagnosis documented on the chart?

Empyema

What theory is used to describe the cause of endometriosis?

Endometrial tissue passes through the Fallopian tubes and into the peritoneal cavity and remains responsive to hormones

A 36-year-old male complains of pain and weakness in the elbow. He reports that he is a warehouse worker and lifts boxes daily. MRI reveals inflammation of the tendon where it attaches to bone. This condition is called:

Epicondylopathy

If a patient had a problem with the adrenal medulla, which of the following hormones should the nurse monitor?

Epinephrine

Stress induces sympathetic stimulation of the adrenal medulla. This causes the secretion of catecholamines, which include: Cortisol and aldosterone Epinephrine and aldosterone Norepinephrine and cortisol Epinephrine and norepinephrine

Epinephrine and norepinephrine

A 25-year-old female presents with burning urination. She was diagnosed with a urinary tract infection. When the nurse checks the culture results, which of the following organisms is most likely infecting her urinary tract?

Escherichia coli

The most common clinical manifestation of portal hypertension is ___bleeding.

Esophageal

Which patient is most prone to metabolic alkalosis? A patient with:

Excessive loss of chloride (Cl)

A nurse is preparing to teach the staff about asthma. Which information should the nurse include? Airway hyper-responsiveness in asthma is related to:

Exposure to an allergen causing mast cell degranulation

A 15-year-old male was struck by a motor vehicle and suffered a traumatic brain injury. Paramedics found him unconscious at the scene of the accident. During the ambulance ride, he regained consciousness and was able to maintain a conversation with the medical staff. Upon arrival to hospital, he was alert and oriented. Physical exam reveals confusion and impaired responsiveness. What is the probable nature of his brain injury based on this history?

Extradural (epidural) hematoma

Which condition is considered a clinical cause of amenorrhea?

Failure to ovulate

Chronic gastritis of the fundus occurs more frequently than chronic gastritis of the antrum.

False

Hepatitis A is transmitted through infected blood and other body fluids

False

Primary biliary cirrhosis can be a result of gallstones

False

Primary biliary cirrhosis can be a result of gallstones.

False

The tonic phase of a generalized seizure is characterized by alternating contractions and relaxation of muscles.

False

A 12-year-old male is newly diagnosed with type 1 DM. What tests should the nurse prepare the patient to best confirm the diagnosis?

Fasting plasma glucose levels

A 12-year-old male is newly diagnosed with type 1 DM. Which of the following tests should the nurse prepare the patient to best confirm the diagnosis?

Fasting plasma glucose levels

A 32-year-old obese male begins a jogging routine. A week after beginning, he fractures his leg. This is referred to as a what type of fracture?

Fatigue

Which are the early (prodromal) clinical manifestations of hepatitis?

Fatigue, fever, hyperalgia, and vomiting

Hypernatremia

Fever, swollen dry tongue, sticky mucous membranes, hallucinations, lethargy, restlessness, irritability, seizures, tachycardia, hypertension, hyper reflexia, twitching, pulmonary edema

A 24-year-old female is diagnosed with renal calculus that is causing obstruction. Which of the following symptoms would she most likely experience?

Flank pain

A 55-year-old male presents reporting urinary retention. Tests reveal that he has a lower urinary tract obstruction. Which of the following is of most concern to the nurse?

Formation of renal calculi

A 25-year-old male was in an automobile accident. At impact, his forehead struck the windshield. In this situation, a nurse recalls the coup injury would occur in the _____ region.

Frontal

Which neurotransmitter is reduced in people with schizophrenia?

Gamma-aminobutyric acid

Which statement is FALSE regarding the contributing factors of duodenal ulcers?

Gastric emptying is slowed causing greater exposure of the mucosa to acid

Which statement is false regarding the contributing factors of duodenal ulcers?

Gastric emptying is slowed causing greater exposure of the mucosa to acid.

If a nurse wants to obtain the best estimate of renal function, which test should the nurse monitor?

Glomerular filtration rate (GFR)

A 30-year-old male is diagnosed with a hormone-secreting tumor of the pancreas alpha cells. Which of the following would the nurse expect to be most likely increased in this patient?

Glucagon

A 15-year-old male suffered diffuse brain injury after wrecking an all-terrain vehicle. He had momentary confusion and retrograde amnesia after 5 to 10 minutes. His injury could be categorized as:

Grade II

A 65-year-old male suffers a subarachnoid hemorrhage secondary to uncontrolled hypertension. He appears drowsy and confused with pronounced focal neurologic deficits. This condition is grade:

Grade III

A 15-year-old male was diagnosed with pharyngitis. Eight days later he developed acute glomerulonephritis. While reviewing the culture results, which of the following is the most likely cause of this disease?

Group A B-hemolytic streptococcus

A 15-year-old male was diagnosed with pharyngitis. Eight days later he developed acute glomerulonephritis. While reviewing the culture results, which of the following is the most likely cause of this disease?

Group A ß-hemolytic streptococcus

A 10-year-old male was physically abused for the previous 4 years. He reports feeling stressed all the time and frequently gets ill. Which of the following hormones is most likely suppressed in this child?

Growth Hormone

A 10-year-old male was physically abused for the previous 4 years. He reports feeling stressed all the time and frequently gets ill. Which of the following hormones is most likely suppressed in this child? Endorphins Prolactin Cortisol Growth hormone

Growth hormone

Outcomes of laboratory tests include an elevated level of natriuretic peptides. Which organ is the priority assessment?

Heart

A 28-year-old male reports to his primary care provider that he has had a cold for a week and is coughing up bloody secretions. When giving report, what term should the nurse use to describe this condition?

Hemoptysis

While checking the lab results for a patient with Graves disease, the nurse would check the T3 level to be abnormally:

High

A 35-year-old female with Graves disease is admitted to a medical-surgical unit. While the nurse is reviewing the lab tests, which results would the nurse expect to find? A. Low circulating levels of thyroid hormone B. Increases circulation of iodine C. Ectopic secretion of thyroid-stimulating hormone (TSH) D. High levels of circulating thyroid-stimulating antibodies

High levels of circulating thyroid-stimulating antibodies Graves disease results from a form of Type II hypersensitivity in which there is stimulation of the thyroid by autoantibodies directed against the TSH receptor.

Which assessment result would the nurse expect to find associated with a patient diagnosed with Graves disease?

High levels of circulating thyroid-stimulating autoantibodies

Which principle should the nurse include while planning care for a patient with an ADH problem ADH release from the posterior pituitary is stimulated by: Low blood pressure sensed by baroreceptors in the kidneys High serum osmolarity sensed by osmoreceptors in the hypothalamus Low osmolality sensed by osmoreceptors in the kidneys High concentration of potassium sensed by chemoreceptors in the carotid body

High serum osmolarity sensed by osmoreceptors in the hypothalamus

Which principle should the nurse include while planning care for a patient with an ADH problem? ADH release from the posterior pituitary is stimulated by:

High serum osmolarity sensed by osmoreceptors in the hypothalamus

A 20-year-old male is admitted to the neurological critical care unit with a severe closed head injury. When an intraventricular catheter is inserted, the ICP is recorded at 24 mm Hg. How should the nurse interpret this reading? This reading is:

Higher than normal

A 15-year-old male is brought to the ER for treatment of injuries received in a motor vehicle accident. An MRI reveals spinal cord injury, and his body temperature fluctuates markedly. The most accurate explanation of this phenomenon is that:

His sympathetic nervous system has been damaged and thermal control disturbed.

Which of these causes condylomata acuminata or genital warts?

Human papillomavirus (HPV)

What causes the chronic microvascular and macrovascular complications of DM?

Hyperglycemia

When a staff member asks the nurse what causes the chronic complications of DM such as microvascular and macrovascular disease, how should the nurse respond? These complications are primarily related to:

Hyperglycemia

When a patient asks what causes hyperglycemia in type 2 DM, how should the nurse respond? Hyperglycemia is a result of:

Hyperinsulinemia and insulin resistance

Which of the following alterations would the nurse expect to find in a patient with untreated Cushing disease or syndrome?

Hypertension

Which of the following alterations would the nurse expect to find in a patient with untreated Cushing disease or syndrome? A. Hyperkalemia B. Bradycardia C. Hypertension D. Tachypnea

Hypertension

Hyperlipidemia and hyperglycemia are associated with:

Hypertonic hyponatremia

For a patient experiencing metabolic acidosis, the body will compensate by:

Hyperventilating

For a patient experiencing metabolic acidosis, the body will compensate by: Hyperventilating Excreting H+ through the kidneys Secreting aldosterone Retaining CO2 in the lungs

Hyperventilating

A 20-year-old male is in acute pain. An arterial blood gas reveals decreased carbon dioxide (CO2) levels. Which of the following does the nurse suspect is the most likely cause?

Hyperventilation

A patient diagnosed with Addison disease reports weakness and is easily fatigued. What is the root of these symptoms?

Hypocortisolism

A patient with Addison disease has weakness and easy fatigability. A nurse recalls this is due to:

Hypocortisolism

A patient with Addison disease has weakness and easy fatigability. A nurse recalls this is due to: A. Hyperkalemia B. Hypocortisolism C. Hypoglycemia D. Metabolic acidosis

Hypocortisolism

A 12-year-old male who uses insulin to control his type 1 diabetes experiences hunger, lightheadedness, tachycardia, pallor, headache, and confusion during gym class. The most probable cause of these symptoms is:

Hypoglycemia caused by increased exercise

A 13-year-old male who uses insulin to control his type 1 diabetes experiences hunger, lightheadedness, tachycardia, pallor, headache, and confusion during gym class. The most probable cause of these symptoms is:

Hypoglycemia caused by increased exercise

A 50-year-old female presents with lightheadedness and overall abnormal feelings. Hyperaldosteronism is diagnosed. Which of the following symptoms would the nurse expect?

Hypokalemia

A 50-year-old female presents with lightheadedness and overall abnormal feelings. Hyperaldosteronism is diagnosed. Which of the following symptoms would the nurse expect? A. Hypokalemia B. Hypotension C. Hyponatremia D. Hypovolemia

Hypokalemia

Which symptom would the nurse expect in a patient diagnosed with hyperaldosteronism?

Hypokalemia

A 54-year-old patient with pulmonary tuberculosis (lung infection) is evaluated for syndrome of inappropriate ADH secretion (SIADH). Which of the following electrolyte imbalances would be expected in this patient? A. Hypernatremia B. Hyperkalemia C. Hyponatremia D. Hypokalemia

Hyponatremia

A 54-year-old patient with pulmonary tuberculosis is evaluated for syndrome of inappropriate ADH secretion (SIADH). Which electrolyte imbalance would be expected in this patient?

Hyponatremia

A 54-year-old patient with pulmonary tuberculosis is evaluated for syndrome of inappropriate ADH secretion (SIADH). Which of the following electrolyte imbalances AND assessments would be expected in this patient?

Hyponatremia AND concentrated urine

A 45-year-old female with Graves disease underwent surgical removal of her thyroid gland. During the postoperative period, her serum calcium was low. The most probable reason for her low serum calcium is:

Hypoparathyroidism caused by surgical injury to the parathyroid glands

When the endocrinologist asks the staff how the releasing hormones that are made in the hypothalamus travel to the anterior pituitary, how should the staff reply? Via the:

Hypophysial portal system

Which of the following would you, the RN, NOT expect to find in a patient with untreated Cushing disease or syndrome?

Hypotension

Hypermagnesemia

Hypotension, drowsiness, bradycardia, Bradypnea, coma, cardiac arrest, hyporeflexia, nausea, vomiting, facial flushing

A male patient complains of tiring easily, has difficulty rising from a sitting position, and cannot stand on his toes. The nurse would expect a diagnosis of:

Hypotonia

A nurse is reviewing the results of an ABG and finds reduced oxygenation of arterial blood. What term should the nurse use to describe this condition?

Hypoxemia

What is the cause of type 1 diabetes mellitus (DM)?

Immune destruction of the pancreas

An 80-year-old male presents with skeletal pain and tenderness, especially in the hips. He is diagnosed with osteomalacia caused by:

Inadequate bone mineralization

A 45-year-old female presents with hypertension, anorexia, nausea and vomiting, and anemia. She is diagnosed with chronic renal failure. When the patient asks what caused this anemia, how should the nurse respond? Your anemia is caused by:

Inadequate production of erythropoietin

Anemia accompanies chronic renal failure because of:

Inadequate production of erythropoietin

In Parkinson disease (PD) the basal ganglia influences the hypothalamic function to produce which grouping of clinical manifestations?

Inappropriate diaphoresis, orthostatic hypotension, constipation, and urinary retention

The common property among the three types of medications used to treat depression is that they do what?

Increase neurotransmitter levels within the synapse

When taking care of a patient with hyperkalemia, which principle is priority? Hyperkalemia causes a(n) _____ in resting membrane potential with _____ excitability of cardiac muscle.

Increase; increased

Which of the following patients is most prone to hypochloremia? A patient with:

Increased bicarbonate intake

Which statement by the staff indicates teaching was successful concerning aldosterone? Secretion of aldosterone results in:

Increased blood volume

Which statement by the staff indicates teaching was successful concerning aldosterone? Secretion of aldosterone results in: Increased blood volume Increased serum potassium levels Decreased plasma osmolality Localized edema

Increased blood volume

A 70-year-old male with chronic renal failure presents with edema. Which of the following is the most likely cause of this condition?

Increased capillary hydrostatic pressure

A 70-year-old male with chronic renal failure presents with edema. Which of the following is the most likely cause of this condition? Decreased interstitial oncotic pressure Increased capillary oncotic pressure Increased interstitial hydrostatic pressure Increased capillary hydrostatic pressure

Increased capillary hydrostatic pressure

An experiment was designed to test the effects of the Starling forces on fluid movement. Which of the following alterations would result in fluid moving into the interstitial space?

Increased interstitial oncotic pressure

An experiment was designed to test the effects of the Starling forces on fluid movement. Which of the following alterations would result in fluid moving into the interstitial space? Increased capillary oncotic pressure Increased interstitial oncotic pressure Increased interstitial hydrostatic pressure Decreased capillary hydrostatic pressure

Increased interstitial oncotic pressure

A nurse is teaching the staff about antidiuretic hormone (ADH). Which information should the nurse include? Secretion of ADH is stimulated by: Generalized edema Increased plasma osmolality Increased serum potassium Decreased renal blood flow

Increased plasma osmolaity

A nurse is teaching the staff about antidiuretic hormone (ADH). Which information should the nurse include? Secretion of ADH is stimulated by:

Increased plasma osmolality

A nurse is teaching the staff about antidiuretic hormone (ADH). Which information should the nurse include? Secretion of ADH is stimulated by: Increased plasma osmolality Increased serum potassium Decreased renal blood flow Generalized edema

Increased plasma osmolality

A nurse is reviewing lab results. Which of the following lab results would slow down the rate of parathyroid hormone secretion?

Increased serum calcium levels

A 50-year-old male patient is deficient in ADH production. What assessment finding would the nurse expect to find?

Increased urine volume

A 50-year-old male patient is deficient in ADH production. Which of the following assessment findings would the nurse expect to find?

Increased urine volume

Which statement is false about the pathophysiology of alcoholic cirrhosis?

Inflammation and damage leading to cirrhosis begin in the bile canaliculi.

The progress notes read: the cerebellar tonsil has shifted through the foramen magnum due to increased pressure within the posterior fossa. The nurse would identify this note as a description of _____ herniation.

Infratentorial

A 57-year-old male presents with cough, sputum production, dyspnea, and decreased lung volume. He is diagnosed with pneumoconiosis. When taking the patient's history, which finding is the most probable cause of his illness?

Inhalation of silica, asbestos, mica

A 19-year-old female with type 1 DM was admitted to the hospital with the following lab values: serum glucose 500 milligrams per deciliter (high), urine glucose and ketones 4+ (high), and arterial pH 7.20 (low). Her parents state that she has been sick with the "flu" for a week. Which of the following statements best explains her acidotic state?

Insulin deficiency promotes lipid metabolism and ketone formation.

When the nurse is asked what causes asthma, how should the nurse respond? Asthma is thought to be caused by:

Interactions between genetic and environmental factors

An infant is diagnosed with noncommunicating hydrocephalus. What is an immediate priority concern for this patient?

Interstitial edema

A 5-year-old male presents to the ER with delirium and sunken eyes. After diagnosing him with severe dehydration, the primary care provider orders fluid replacement. The nurse administers a hypertonic intravenous solution. Which of the following would be expected?

Intracellular dehydration

A 35-year-old female was severely burned and is hospitalized. She is now suffering from acute tubular necrosis (ATN). Which of the following is the most likely diagnosis the nurse will observe on the chart?

Intrarenal

A nurse is reviewing lab reports. The nurse recalls blood plasma is located in which of the following fluid compartments?

Intravascular fluid

A nurse is reviewing lab reports. The nurse recalls blood plasma is located in which of the following fluid compartments? Intravascular fluid Intracellular fluid (ICF) Extracellular fluid (ECF) Interstitial fluid

Intravascular fluid

A nurse recalls the glomerular filtration rate (GFR) and plasma creatinine (Pcr) concentration are _____ related.

Inversely

Which nutrient would the nurse encourage the patient to consume for thyroid hormone synthesis?

Iodine

For a patient with respiratory acidosis, chronic compensation by the body will include:

Kidney excretion of H+

For a patient with respiratory acidosis, chronic compensation by the body will include: Kidney excretion of HCO3 Kidney excretion of H+ Protein buffering Prolonged exhalations to blow off CO2

Kidney excretion of H+

For a patient with respiratory acidosis, chronic compensation by the body will include: Protein buffering Prolonged exhalations to blow off CO2 Kidney excretion of H+ Kidney excretion of HCO3

Kidney excretion of H+

Which organ system should the nurse monitor when the patient has long-term potassium deficits?

Kidneys

A 30-year-old diagnosed with Graves disease is admitted to a hospital unit for the surgical removal of the thyroid gland. During the postoperative period, the nurse notes that the patient's serum calcium is low. The nurse should observe the patient for which of the following signs/symptoms?

Laryngeal spasms and hyperreflexia

A 30-year-old female with Graves disease is admitted to a hospital unit for the surgical removal of her thyroid gland. During the postoperative period, the nurse notes that the patient's serum calcium is low. The nurse should observe the patient for which of the following signs/symptoms? A. Abdominal pain and fever B. Laryngeal spasms and hyperreflexia C. Anorexia, nausea, vomiting D. Muscle weakness and constipation

Laryngeal spasms and hyperreflexia

A 30-year-old female with Graves disease is admitted to a hospital unit for the surgical removal of her thyroid gland. During the postoperative period, the nurse notes that the patient's serum calcium is low. The nurse should observe the patient for which of the following signs/symptoms?

Laryngeal spasms, tetany and hyperreflexia

A nurse is teaching staff about pulmonary edema. Which information should the nurse include? The most common cause of pulmonary edema is:

Left heart failure

Which patient would the nurse assess for paroxysmal nocturnal dyspnea (PND)? A patient with:

Left ventricular failure

Which of the following patients should the nurse assess for a decreased oncotic pressure in the capillaries? A patient with:

Liver failure

Which of the following patients should the nurse assess for a decreased oncotic pressure in the capillaries? A patient with: A high-protein diet Low blood pressure Low blood glucose Liver failure

Liver failure

A 10-year-old male is brought to the emergency room (ER) because he is incoherent and semiconscious. CT scan reveals that he is suffering from cerebral edema. This type of edema is referred to as:

Localized edema

A 10-year-old male is brought to the emergency room (ER) because he is incoherent and semiconscious. CT scan reveals that he is suffering from cerebral edema. This type of edema is referred to as: Lymphedema Generalized edema Pitting edema Localized edema

Localized edema

A 10-year-old male is brought to the emergency room (ER) because he is incoherent and semiconscious. CT scan reveals that he is suffering from cerebral edema. This type of edema is referred to as: Pitting edema Localized edema Generalized edema Lymphedema

Localized edema

A 53-year-old male with a 20-year history of smoking is diagnosed with emphysema. When a staff member asks why the patient's airways are obstructed, how should the nurse respond? The airways are obstructed because of:

Loss of elastic recoil

A nurse thinks a patient may be experiencing dementia. Which assessment finding will most help support this diagnosis?

Loss of recent and remote memory

Dementia is characterized by:

Loss of recent and remote memory

While planning care for a patient with hypothyroidism, which principle should the nurse remember? The basal metabolic rate is unusually _____ with hypothyroidism.

Low

While planning care for a patient with hypothyroidism, which principle should the nurse remember? The basal metabolic rate is unusually _____ with hypothyroidism. A. Steady B. Low C. High D. Variable

Low

A nurse recalls regulation of acid-base balance through removal or retention of volatile acids is accomplished by the:

Lungs

Hypothalamic-pituitary-adrenal (HPA) system abnormalities exist in a large percentage of individuals with what?

Major depression

Which of the following people is at highest risk for the development of gout?

Men aged 40 to 50 years

A 39-year-old male suffers a severe brain injury when he falls off a building while working. CT scan reveals that he has a basilar skull fracture. Based upon his injuries, what major complication should the nurse observe for in this patient?

Meningeal Infection

A 3-year-old male was diagnosed with congenital hypothyroidism. The parents ask the nurse if left untreated what will happen. What is the nurse's best response? If left untreated, the child would have: A. Liver, kidney, and pacreas failure B. Hyperactivity and ADD C. Mental retardation and stunted growth D. Increased risk of childhood thyroid cancer

Mental retardation and stunted growth

A 54-year-old male was recently diagnosed with rheumatoid arthritis (RA). Which of the following is the expected treatment of choice?

Methotrexate

A nurse wants to determine if there is kidney dysfunction in a patient with diabetes. Which of the following is the earliest manifestation?

Microalbuminuria

What is the earliest manifestation of diabetes-induced kidney dysfunction?

Microalbuminuria

A 10-year-old male was climbing on a house and fell. He suffered a severe brain injury. His Glasgow Coma Scale (GCS) was 5 initially and 7 after 1 day. He remained unconscious for 2 weeks, then was confused and suffered from anterograde amnesia. Which of the following is he most likely experiencing?

Moderate Diffuse Brain Injury

A nurse is reviewing lab results for glycosylated hemoglobin (Hemoglobin A1c) levels. A nurse recalls the purpose of this test is to:

Monitor long term serum glucose control

A nurse is reviewing lab results for glycosylated hemoglobin (hemoglobin A1c) levels. A nurse recalls the purpose of this test is to:

Monitor long-term serum glucose control.

What is the purpose of the glycosylated hemoglobin (hemoglobin A1c) test?

Monitoring long-term serum glucose control.

A nurse is preparing to teach the staff about asthma. Which information should the nurse include? Airway obstruction contributing to increased airflow resistance and hypoventilation in asthma is caused by:

Mucus secretion, bronchoconstriction, and airway edema

Hypercalcemia

Muscle weakness, hypercalciuria/kidney stones, dysrhythmias, lethargy,coma,hyporeflexia, pathologic fractures,flank pain, deep bone pain, polyuria,polydipsia,dehydration,hypertension, nausea, vomiting

Hypokalemia

Muscle weakness/ cramping, fatigue, nausea, vomiting, irritability, confusion, decreased bowel motility, paresthesia, dysrhythmias, flat/inverted T waves (ECG)

A 25-year-old male presents to his primary care provider reporting changes in facial features. CT scan reveals a mass on the anterior pituitary, and lab tests reveal severely elevated growth hormone (GH). Which of the following would the nurse also expect to find?

Muscular atrophy

When the nurse is discussing the functional unit of the kidney, what other term should the nurse use?

Nephron

A nurse recalls direct stimulation of the insulin-secreting cells of the pancreas by the autonomic nervous system is an example of _____ control

Neural

A 50-year-old male patient presents with polyuria and extreme thirst. He was given exogenous ADH. For what condition would this treatment be effective

Neurogenic diabetes insipidus

A 50-year-old male patient presents with polyuria and extreme thirst. He was given exogenous ADH. For which of the following conditions would this treatment be effective?

Neurogenic diabetes insipidus

A patient presents with polyuria and extreme thirst and is given exogenous ADH. For which of the following conditions would this treatment be effective?

Neurogenic diabetes insipidus

A 50-year-old male patient presents with polyuria and extreme thirst. He was given exogenous ADH. For which of the following conditions would this treatment be effective? A. SIADH B. Neurogenic diabetes insipidus C. Nephrogenic diabetes insipidus D. Psychogenic diabetes insipidus

Neurogenic diabetes insipidus Neurogenic diabetes insipidus is caused by the insufficient secretion of ADH; thus, exogenous ADH would be useful in the treatment of this disorder.

A nurse is caring for a patient diagnosed with SIADH. What severe complication should the nurse assess for?

Neurologic damage

A nurse is caring for a patient with SIADH. What severe complication should the nurse assess for?

Neurologic damage

A nurse is caring for a patient with SIADH. What severe complication should the nurse assess for?

Neurological Damage

Which factor increases the risk for ovarian cancer after the age of 40 years?

Never having children

A 60-year-old male with a 40-year history of smoking presents with chest pain, cough, sputum production, and pneumonia. Tests reveal widespread metastatic cancer, and the primary care provider plans radiation therapy. Which of following is the most likely type of cancer to be documented on the chart?

Non-small cell carcinoma

A 60-year-old female with emphysema is having difficulty expiring a given volume of air. When giving report, the nurse will relay that the patient is most likely experiencing _____ pulmonary disease.

Obstructive

A 60-year-old female is diagnosed with hyperkalemia. Which assessment finding should the nurse expect to observe?

Oliguria

A 20-year-old male presents to his primary care provider reporting difficulty breathing when lying down. What term should the nurse use to document this condition?

Orthopnea

The type of diarrhea that is a result of unhydrolyzed lactose is referred to as:

Osmotic

Water movement between the ICF and ECF compartments is determined

Osmotic forces

Water movement between the ICF and ECF compartments is determined by: Buffer systems Osmotic forces Plasma oncotic pressure Antidiuretic hormone

Osmotic forces

A 49-year-old female is diagnosed with hypercortisolism. What should the nurse expect?

Osteoporosis

A nurse assesses a patient with a complicated urinary tract infection (UTI) for:

Other health problems

A 39-year-old female is recovering from the birth of her third child. What hormone would help prevent uterine bleeding?

Oxytocin

If a patient had a problem with the hypothalamus, which of the following hormones would be affected?

Oxytocin

When assessing the effects of elevated â-endorphins in a patient, which of the following should the nurse monitor? Hyperglycemia Peripheral vasoconstriction Pain inhibition Decreased immune cell activity

Pain inhibition

Pancreatic beta cells secrete insulin, which inhibits secretion of glucagon from neighboring alpha cells. This action is an example of which of the following signaling types?

Paracrine

A patient has paralysis of both legs. What type of paralysis does the patient have?

Paraplegia

What is the most common cause of hypoparathyroidism?

Parathyroid gland injury

When a patient wants to know what most commonly causes hypoparathyroidism, how should the nurse reply? It is most commonly caused by: A. Parathyroid gland injury B. Hypothalamic inactivity C. Pituitary hyposecretion D. Parathyroid adenoma

Parathyroid gland injury

Hypomagnesemia

Paresthesia, dysrhythmias, Trousseau's sign, Chvostek's sign, agitation, confusion, hyperreflexia, hypertension, insomnia,irritability, anorexia, nausea, vomiting, dysphagia

Hypophosphatemia

Paresthesia, muscle weakness, bone pain and deformities, chest pain, confusion,seizures,nystagmus

A nurse is teaching a patient with diabetes how glucose is transported from the blood to the cell. What type of transport system should the nurse discuss with the patient?

Passive-mediated transport (facilitated diffusion

A 76-year-old female was diagnosed with osteoporosis by radiologic exam. She is at high risk for:

Pathologic bone fractures

A 70-year-old female with osteoporosis fractures her leg at a location of preexisting abnormality following a minor fall. Which of the following best describes the fracture?

Pathologic fracture

Hyperkalemia

Peaked T waves, ventricular dysrhythmias, muscle twitching and paresthesia (early), ascending muscle weakness (late), increased bowel motility

A man has balanitis. What action by the healthcare professional is most appropriate?

Perform a finger stick for a blood glucose reading.

A 65-year-old Hispanic female is admitted to the hospital with a pathologic, compound, transverse fracture of the femur. Which of the following statements best describes the location of this fracture?

Perpendicular across the bone

A 30-year-old female presents with hypertension, headache, tachycardia, impaired glucose tolerance, and weight loss. Which of the following diagnosis will the nurse see documented on the chart? A. Cushing disease B. Conn disease C. Addison disease D. Phenochromocytoma

Phenochromocytoma

A 30-year-old female presents with hypertension, headache, tachycardia, impaired glucose tolerance, and weight loss. What diagnosis will the nurse see documented on the chart?

Pheochromocytoma

A 30-year-old female presents with hypertension, headache, tachycardia, impaired glucose tolerance, and weight loss. Which of the following diagnosis will the nurse see documented on the chart?

Pheochromocytoma

A 30-year-old presents with hypertension, headache, tachycardia, impaired glucose tolerance, and weight loss. Which of the following diagnoses is supported by this symptomology?

Pheochromocytoma

A 47-year-old male is diagnosed with pulmonary edema. Which assessment findings will the nurse observe?

Pink, frothy sputum

A 25-year-old male presents with fatigue, constipation, and sexual dysfunction. Tests reveal all pituitary hormones are normal and no masses are present. The nurse suspects the most likely cause of his symptoms is a dysfunction in the: A. Pituitary stalk B. Posterior pituitary C. Anterior pituitary D. Pars intermedia

Pituitary stalk When pituitary hormones are normal, dysfunction in the action of hypothalamic hormones are most commonly related to interruption of the connection between the hypothalamus and pituitary, the pituitary stalk.

The RN is assigned a patient who is one day post-operative thyroidectomy. During AM assessment the RN notes that the dressing is dry and intact and vital signs are stable. The patient complains that he feels tingling around his mouth when he drinks or eats anything. The RN notes facial twitching in response to a light tap over the facial nerve. Which of the following laboratory measurements provide physiological rationale for these symptoms?

Plasma calcium= total(4mg/dl) 3.3 mg/dL(ionized)

A 65-year-old male recently had a cerebrovascular accident that resulted in dysphagia. He now has aspiration of gastric contents. The nurse assesses the patient for which complication?

Pneumonia

Which finding would support the diagnosis of respiratory acidosis?

Pneumonia

An 11-year-old is newly diagnosed with type 1 DM. Which classic symptoms should the nurse assess the patient for?

Polydipsia, polyuria, polyphagia, and weight loss

An 11-year-old male is newly diagnosed with type 1 DM. Which classic symptoms should the nurse assess the patient for?

Polydipsia, polyuria, polyphagia, and weight loss

An 11-year-old male is newly diagnosed with type 1 DM. Which classic symptoms should the nurse assess the patient for? A. Polydipsia, polyuria, weight loss B. Recurrent infections C. Weakness, vomiting, hypotension, mental confusion D. Vomiting, abdominal pain, sweet, fruity breath, and Kussmaul breathing

Polydipsia, polyuria, weight loss

When a staff member asks the nurse which gland secretes ADH and oxytocin, how should the nurse respond?

Posterior pituitary

A 56-year-old male presents with flank pain and polyuria. Tests reveal that he has an enlarged prostate. Which of the following types of renal failure should the nurse monitor for as it is the most likely to occur?

Postrenal

A nurse recalls insulin has an effect on what group of electrolytes?

Potassium, magnesium, phosphate

A nurse recalls insulin has an effect on which of the following groups of electrolytes?

Potassium, magnesium, phosphate

A 60-year-old male undergoes surgery for a bone fracture. Which of the following nursing measures would be most effective for preventing pulmonary embolism (PE) in this patient?

Prevent deep vein thrombosis formation.

A 42-year-old female presents to her primary care provider reporting muscle weakness and cardiac abnormalities. Laboratory tests indicate that she is hypokalemic. Which of the following could be the cause of her condition?

Primary hyperaldosteronism

A 25-year-old male was diagnosed with Goodpasture syndrome. While planning care for this patient, which of the following mechanisms would cause tissue injury?

Production of antibodies against the glomerular basement membrane

A 42-year-old female is diagnosed with chronic renal failure, and the nurse is discussing dietary treatment. Which information indicates the nurse understands dietary regimen? Treatment includes restricting:

Proteins

An 80-year-old female is in the hospital for a bone fracture. While there she develops a large, nonlethal pulmonary embolus. Which of the following is a direct result of the obstruction to pulmonary blood flow?

Pulmonary hypertension

A 25-year-old male presents with chronic bronchitis of 5 months' duration. Which of the following is the most significant concern for the nurse to monitor in this patient?

Recurrent pulmonary infections

Which patient should the nurse assess for both hyperkalemia and metabolic acidosis? A patient diagnosed with:

Renal failure

A 46-year-old male presents with severe pain, redness, and tenderness in the right big toe. He was diagnosed with gouty arthritis. He is at risk for developing:

Renal stones

The most common cause of pure water deficit is:

Renal water loss

A 70-year-old female presents with a hip fracture secondary to osteoporosis. This condition is caused by an increase in bone:

Resorption

A 54-year-old male with a long history of smoking complains of excessive tiredness, shortness of breath, and overall ill feelings. Lab results reveal decreased pH, increased CO2, and normal bicarbonate ion. These findings help to confirm the diagnosis of:

Respiratory acidosis

A 55-year-old female presents to her primary care provider and reports dizziness, confusion, and tingling in the extremities. Blood tests reveal an elevated pH, decreased PCO2, and slightly decreased HCO3. Which of the following is the most likely diagnosis?

Respiratory alkalosis with renal compensation

A 55-year-old female presents to her primary care provider and reports dizziness, confusion, and tingling in the extremities. Blood tests reveal an elevated pH, decreased PCO2, and slightly decreased HCO3. Which of the following is the most likely diagnosis? Respiratory alkalosis with renal compensation Respiratory acidosis with renal compensation Metabolic acidosis with respiratory compensation Metabolic alkalosis with respiratory compensation

Respiratory alkalosis with renal compensation

Which assessment finding by the nurse characterizes a mild concussion?

Retrograde Amnesia

A 21-year-old female presents with low back pain and stiffness that is alleviated by physical activity. She was diagnosed with ankylosing spondylitis (AS). The first joint to be affected would be the:

Sacroiliac

Which hypothyroid diagnosis is supported by low levels of TSH?

Secondary

A person who has cholera would be expected to have which type of diarrhea?

Secretory diarrhea

A decrease in receptor binding for which neurotransmitter is found in individuals with depression?

Serotonin

A nurse is teaching a patient about insulin. Which information should the nurse include? Insulin is primarily regulated by:

Serum glucose levels

A 25-year-old male presents to his primary care provider reporting changes in facial features. CT scan reveals a mass on the anterior pituitary and lab tests reveal several elevated growth hormone (GH). What would the nurse expect to find?

Sexual dysfunction

When a patient has a massive pulmonary embolism (PE), what complications will the nurse monitor for?

Shock and death

A nurse remembers a low ventilation-perfusion ratio results in:

Shunting

A 35-year-old female with Graves disease is admitted to a medical-surgical unit. Which of the following symptoms would the nurse expect to find before treatment?

Skin hot and moist, rapid heart rate

A 35-year-old female with Graves disease is admitted to a medical-surgical unit. Which of the following symptoms would the nurse expect to find before treatment? A. Constipation, confusion B. Skin hot and moist, rapid heart rate C. Slow heart rate, rash D. Weight gain, cold intolerance

Skin hot and moist, rapid heart rate

A patient diagnosed with Graves disease is admitted to a medical-surgical unit. Which of the following symptoms would the nurse expect to find before treatment?

Skin hot and moist, rapid heart rate

In the majority of cases delayed puberty is due to:

Slow maturation

A patient has been searching on the Internet about natriuretic hormones. When the patient asks the nurse what do these hormones do, how should the nurse respond? Natriuretic hormones affect the balance of:

Sodium

A patient has been searching on the Internet about natriuretic hormones. When the patient asks the nurse what do these hormones do, how should the nurse respond? Natriuretic hormones affect the balance of: Sodium Magnesium Calcium Potassium

Sodium

When planning care for a dehydrated patient, the nurse remembers the principle of water balance is closely related to _____ balance.

Sodium

When planning care for a dehydrated patient, the nurse remembers the principle of water balance is closely related to _____ balance. Potassium Sodium Bicarbonate Chloride

Sodium

A 12-year-old female hurts her ankle while playing basketball. Tests reveal that she tore a ligament. This condition is known as a:

Sprain

A 70-year-old female is being closely monitored in the neurological critical care unit for a severe closed head injury. After 48 hours, her condition begins to deteriorate. Her pupils are small and sluggish, pulse pressure is widening, and she is bradycardic. These clinical findings are evidence of what stage of intracranial hypertension?

Stage 3 Early decompensated

A 54-year-old male is diagnosed with empyema. Upon receiving and reviewing the culture result, which organism does the nurse suspect is the most likely cause?

Staphylococcus aureus

A 10-year-old male is brought to the ER with prolonged bronchospasm and severe hypoxemia. The most likely diagnosis on the chart is:

Status asthmaticus

A patient wants to know what can cause ACTH to be released. How should the nurse respond?

Stress

A 75-year-old male reports to his primary care provider loss of urine with cough, sneezing, or laughing. Which of the following is the most likely diagnosis the nurse will observe on the chart?

Stress incontinence

A 48-year-old female presents at the ER reporting an acute severe headache, nausea, photophobia, and nuchal rigidity. What does the nurse suspect caused these signs and symptoms?

Subarachnoid Hemorrhage

A 35-year-old female suffers a broken clavicle following a motor vehicle accident. X-ray reveals that the bone surfaces in the joint partially lost contact with each other. This condition is called:

Subluxation

When a patient asks how bad the injuries will be from a spinal injury, what is the nurse's best response? It is difficult to know the full extent of the injury because of:

Swelling within the spinal cord

Which of the following concerning Gestational diabetes is incorrect?

Symptoms appear in the first trimester of pregnancy

A 34-year-old female was recently diagnosed with RA. Physical examination revealed that inflammation started in the:

Synovial membrane

A patient has positive signs of schizophrenia and is scheduled for a brain scan. What part of the brain does the healthcare professional expect the scan will focus on?

Temporal lobe

A 50-year-old male presents with hypotension, hypoxemia, and tracheal deviation to the left. Tests reveal that the air pressure in the pleural cavity exceeds barometric pressure in the atmosphere. Based upon these assessment findings, what does the nurse suspect the patient is experiencing?

Tension pneumothorax

In which stage of syphilis would the following clinical manifestations be found: destructive skin, bone and soft tissue lesions, aneurysms, heart failure, and neurosyphilis?

Tertiary

The risk of which cancer is greater if the man has a history of cryptorchidism?

Testicular

Cryptorchidism can be defined as which of the following?

Testicular maldescent

Hyperphosphatemia

Tetany, cramps, paresthesia, dysrhythmias, Trousseau's sign, Chvostek's sign, hyperreflexia, anorexia, nausea, vomiting, soft tissue calcification

Hypocalcemia

Tetany,cramps,paresthesia, dysrhythmias, Trousseau's sign, Chvostek's sign, seizures,hyperreflexia, impaired clotting time

The primary care provider states that the patient has vasogenic edema. The nurse realizes vasogenic edema is clinically important because:

The blood-brain barrier is disrupted.

A patient wants to know why ADH is important in the body. What is the nurse's best response ADH is important in: The body's water balance and urine concentration Maintaining electrolyte levels and concentrations Follicular maturation Regulation of metabolic processes

The body's water balance and urine concentration

A patient wants to know why ADH is important in the body. What is the nurse's best response? ADH is important in:

The body's water balance and urine concentration

A patient wants to know why ADH is important in the body. What is the nurses best response? ADH is important in:

The body's water balance and urine concentration

_____ are most at risk of spinal cord injury from minor trauma.

The elderly

While turning a patient with chronic renal failure, which principle should the nurse recall? Bone fractures are a risk factor in chronic renal failure because:

The kidneys fail to activate vitamin D.

When a nurse is checking the urinalysis, plasma proteins should be absent from the urine because:

The negative charge of the glomerular filtration membrane repels the plasma proteins

An endocrinologist is teaching about aldosterone secretion. Which information should the endocrinologist include? Aldosterone secretion is regulated by:

The renin-angiotensin system

A healthcare professional is teaching a high school group about STIs. Which statement by the professional provides the most accurate information regarding the transmission of herpes simplex virus (HSV)?

The risk of transmission is present even during latent periods

A 22-year-old female presents with chronic bronchitis. Tests reveal closure of the airway during expiration. While planning care, a nurse recalls this condition is most likely caused by:

Thick mucus from hypertrophied glands

A 45-year-old male presents with oliguria. He is diagnosed with chronic glomerulonephritis. The nurse knows oliguria is related to:

Thickening of the glomerular membrane and decreased renal blood flow

Diabetes insipidus, diabetes mellitus (DM), and SIADH share which of the following assessment manifestations?

Thirst

Diabetes insipidus, diabetes mellitus (DM), and SIADH share which of the following assessment manifestations? A. Edema B. Polyuria C. Vomiting and abdominal cramping D. Thirst

Thirst

The most common manifestation of portal hypertension induced splenomegaly is:

Thrombocytopenia

A 30-year-old male was diagnosed with hypothyroidism. Synthesis of what would decrease in this patient?

Thyroid-binding globulin

The final stage of gout, characterized by crystalline deposits in cartilage, synovial membranes, and soft tissue, is called:

Tophaceous gout

A 60-year-old female with a history of cirrhosis presents with dyspnea, impaired ventilation, and pleural pain. A diagnosis of pleural effusion is made, and a watery fluid is drained. When giving report, the nurse will refer to this fluid as:

Transudative

A 51-year-old male experienced severe acute gouty arthritis. Which of the following is the most common trigger for the symptoms?

Trauma

When a nurse is assessing the physical features of individuals with Cushing syndrome, these findings will include:

Truncal obesity and moon face

When a nurse is assessing the physical features of individuals with Cushing syndrome, these findings will include: A. Pallor and swollen tongue B. Truncal obesity and moon face C. Weight loss and muscle wasting D. Depigmented skin and eyelid lag

Truncal obesity and moon face

Which physical feature supports the diagnosis of Cushing syndrome?

Truncal obesity and moon face

A 35-year-old female took corticosteroid therapy for several months. Which of the following would the nurse expect to find?

Type 2 DM

A 35-year-old female took corticosteroid therapy for several months. Which of the following would the nurse expect to find? A. Type 2 DM B. Renal toxicity C. Episodes of hypoglycemia D. Hypotension

Type 2 DM

A 46-year-old male presents with severe pain, redness, and tenderness in the right big toe. He was diagnosed with gouty arthritis. The symptoms he experienced are caused by the crystallization of _____ within the synovial fluid.

Uric acid

While planning care for a patient who has acute pyelonephritis. A nurse recalls the most common condition associated with the development of acute pyelonephritis is:

Urinary tract obstruction

While planning care for a patient with renal calculi, the nurse remembers the most important factor in renal calculus formation is:

Urine pH

What type of cerebral edema occurs when there is increased permeability of capillary endothelium after injury to the vascular structure?

Vasogenic

A patient asks what causes pneumonia. How should the nurse reply? Pneumonia is caused by:

Viral or bacterial infections

What common neurologic disturbances should the nurse assess for in a patient with pituitary adenoma?

Visual distrubances

What common neurologic disturbances should the nurse assess for in a patient with a pituitary adenoma?

Visual disturbances

What common neurologic disturbances should the nurse assess for in a patient with a pituitary adenoma? A Confused states B. Visual disturbances C. Coma D. Breathing abnormalities

Visual disturbances The clinical manifestations of pituitary adenomas are visual changes including visual field impairments (often beginning in one eye and progressing to the other) and temporary blindness.

A 56-year-old male was admitted to the hospital with a diagnosis of osteomalacia. History reveals that he takes anticonvulsants, underwent small bowel resection 3 years earlier, and suffers from chronic pancreatitis. What is the common link between these three factors and the development of osteomalacia?

Vitamin D deficiency

Hyponatremia

Weakness, lethargy, confusion, seizures, headache, anorexia, nausea, vomiting, muscle cramps, twitching, hypotension, tachycardia, weight gain, edema

The nurse would anticipate the patient with syndrome of inappropriate ADH (SIADH) to demonstrate which of the following symptoms?

Weakness, nausea, headache, and muscle twitching

The nurse would anticipate the patient with syndrome of inappropriate ADH (SIADH) to demonstrate which of the following symptoms? (Select all that apply.)

Weakness, nausea, muscle twitching, headache, and weight gain

What is the first sign of puberty in girls? a. Breast enlargement b. Growth of pubic hair c. Menstruation d. Vaginal discharge

a. Breast enlargement

Which virus is a precursor for developing cervical intraepithelial neoplasia (CIN) and cervical cancer? a. Human papillomavirus (HPV) b. Epstein-Barr virus (EBV) c. Herpes simplex II virus (HSV) d. Cytomegalovirus (CMV)

a. Human papillomavirus (HPV)

Which benign breast tumor affects postmenopausal women and is characterized by the principal lactiferous ducts becoming dilated and filled with cellular debris? a. Mammary duct ectasia b. Intraductal papilloma c. Phyllodes tumor d. Fibroadenoma

a. Mammary duct ectasia

Which type of precocious puberty causes the child to develop some secondary sex characteristics of the opposite sex? a. Mixed b. Incomplete c. Isosexual d. Homosexual

a. Mixed

Considering the pathophysiologic characteristics of primary amenorrhea, what anatomic structure is involved in compartment II? a. Ovary b. Anterior pituitary c. Hypothalamus d. Vagina

a. Ovary

What is a recognized treatment for the symptoms often associated with pelvic organ prolapse? (Select all that apply.) a. Pessary b. Kegel exercises c. Estrogen therapy d. Surgical repair e. Bearing down exercises

a. Pessary b. Kegel exercises c. Estrogen therapy d. Surgical repair

The size of benign uterine tumors, such as leiomyomas, is thought to be caused by the influence of which hormone? (Select all that apply.) a. Progesterone b. Estrogen c. Luteinizing hormone d. Gonadotropin-stimulating hormone e. Growth factors

a. Progesterone b. Estrogen e. Growth factors

Which of the following is not considered a cause of galactorrhea? a. Proliferation of the lactiferous ducts of the breast b. Hypothyroidism, resulting from a decrease in thyroid-releasing hormone c. Excess prolactin secretion from the pituitary d. Drugs such as high-dose oral contraceptives and phenothiazines

a. Proliferation of the lactiferous ducts of the breast

The majority of the small percentage of ovarian cancers that are associated with a known pattern of inheritance are associated with: a. Susceptibility of the BRCA1 gene b. Mutations of the BRCA2 gene c. Hereditary nonpolyposis colorectal cancer (HNPCC) syndrome d. Low progesterone levels

a. Susceptibility of the BRCA1 gene

Which factor increases the risk for ovarian cancer after the age of 40 years? a. Use of fertility drugs b. Oral contraceptive use c. Multiple pregnancies d. Prolonged lactation

a. Use of fertility drugs

Physiologic pH is maintained around 7.4 because carbonic acid and bicarbonate exist in a ratio of: a. 20:1 b. 1:20 c. 10:2 d. 2:10

a. 20:1

When taking care of a patient with hyperkalemia, which principle is priority? Hyperkalemia causes a(n) _____ in resting membrane potential with _____ excitability of cardiac muscle. a. Increase; increased b. Decrease; increased c. Increase; decreased d. Decrease; decreased

a. Increase; increased

For a patient with respiratory acidosis, chronic compensation by the body will include: a. Kidney excretion of H+ b. Kidney excretion of HCO3 c. Prolonged exhalations to blow off CO2 d. Protein buffering

a. Kidney excretion of H+ Rationale: The kidneys excrete H+ to compensate for respiratory acidosis.

A 10-year-old male is brought to the emergency room (ER) because he is incoherent and semiconscious. CT scan reveals that he is suffering from cerebral edema. This type of edema is referred to as: a. Localized edema b. Generalized edema c. Pitting edema d. Lymphedema

a. Localized edema Rationale: Cerebral edema is a form of localized edema. Generalized edema is manifested by a more uniform distribution of fluid in interstitial spaces. Pitting edema is due to a pit left in the skin. Lymphedema is due to swelling in interstitial spaces, primarily in the extremities.

Water movement between the ICF and ECF compartments is determined by: a. Osmotic forces b. Plasma oncotic pressure c. Antidiuretic hormone d. Buffer systems

a. Osmotic forces

The most common cause of pure water deficit is: a. Renal water loss b. Hyperventilation c. Sodium loss d. Insufficient water intake

a. Renal water loss Rationale: The most common cause of water loss is increased renal clearance of free water as a result of impaired tubular function.

A 55-year-old female presents to her primary care provider and reports dizziness, confusion, and tingling in the extremities. Blood tests reveal an elevated pH, decreased PCO2, and slightly decreased HCO3. Which of the following is the most likely diagnosis? a. Respiratory alkalosis with renal compensation b. Respiratory acidosis with renal compensation c. Metabolic alkalosis with respiratory compensation d. Metabolic acidosis with respiratory compensation

a. Respiratory alkalosis with renal compensation Rationale: With an elevated pH, the diagnosis must be alkalosis. Since the PCO2 is low, it is likely respiratory with a slight decrease in HCO3 indicating renal compensation.

A nurse is discussing endocrine system dysfunction with a patient. What statement indicated that patient understood? Endocrine system dysfunction can result from hyposecretion, hypersecretion, or from:

abnormal receptor activity

Besides hyposecretion and hypersecretion, endocrine system dysfunction can result from:

abnormal receptor activity.

The nurse is teaching the staff about aldosterone. Which information should the nurse include The main site of aldosterone synthesis is the: Liver Kidneys Adrenal cortex Hypothalamus

adrenal cortex

A nurse checks lab results as both Cushing disease and Addison disease can manifest with elevated levels of:

adrenocorticotropic hormone (ACTH).

A 50-year-old male with one kidney had to undergo surgery for an adrenal tumor. His zona glomerulosa was largely removed during the surgery. The nurse would expect the removal of this tumor to result in a decrease in: Sodium Aldosterone Potassium Acid

aldosterone

A nurse recalls prolactin-inhibiting factor's target tissue is the: Hypothalamus Anterior pituitary Mammary glands Posterior pituitary

anterior pituitary

A 55-year-old female is admitted to the medical unit for complications of long-term, poorly controlled type 2 DM. What should the nurse expect to find in addition to elevated glucose?

atherosclerosis

Infertility is defined as the inability to conceive after how many months of unprotected intercourse with the same partner? a. 6 b. 12 c. 18 d. 24

b. 12

Clinical manifestations that include irregular or heavy bleeding, the passage of large clots, and the depletion of iron stores support which diagnosis? a. Premenstrual syndrome b. Dysfunctional uterine bleeding c. Polycystic ovary syndrome d. Primary dysmenorrhea

b. Dysfunctional uterine bleeding

What theory is used to describe the cause of endometriosis? a. Obstruction within the fallopian tubes prevents the endometrial tissue from adhering to the lining of the uterus. b. Endometrial tissue passes through the fallopian tubes and into the peritoneal cavity and remains responsive to hormones. c. Inflammation of the endometrial tissue develops after recurrent sexually transmitted diseases. d. Endometrial tissue lies dormant in the uterus until the ovaries produce sufficient hormone to stimulate its growth.

b. Endometrial tissue passes through the fallopian tubes and into the peritoneal cavity and remains responsive to hormones.

Considering the pathophysiologic characteristics of primary amenorrhea, what anatomic structure is involved in compartment IV? a. Vagina b. Hypothalamus c. Ovary d. Anterior pituitary

b. Hypothalamus

What are the common clinical manifestations of endometriosis? (Select all that apply.) a. Back and flank pain b. Infertility c. Dysuria d. Amenorrhea e. Dysmenorrhea

b. Infertility e. Dysmenorrhea

Dysfunctional uterine bleeding (DUB), secondary to ovarian dysfunction, is abnormal uterine bleeding resulting from: (Select all that apply.) a. Endometriosis b. Progesterone deficiency c. Sexually transmitted infections d. Congenital abnormalities in the uterine structure e. Estrogen excess

b. Progesterone deficiency e. Estrogen excess

The release of which chemical mediator causes primary dysmenorrhea? a. Leukotrienes b. Prostaglandins c. Bradykinin d. C-reactive protein

b. Prostaglandins

A 19-year-old male presents to his primary care provider reporting restlessness, muscle cramping, and diarrhea. Lab tests reveal that he is hyperkalemic. Which of the following could have caused his condition? a. Primary hyperaldosteronism b. Acidosis c. Insulin secretion d. Diuretic use

b. Acidosis Rationale: During acute acidosis, hydrogen ions accumulate in the ICF and potassium shifts out of the cell to the ECF, causing hyperkalemia. Primary hyperaldosteronism is associated with hypokalemia, not hyperkalemia. Insulin secretion helps reduce potassium levels in the cell, not cause it. Diuretics would cause hypokalemia, not hyperkalemia.

Which of the following conditions would cause the nurse to monitor for hyperkalemia? a. Excess aldosterone b. Acute acidosis c. Insulin usage d. Metabolic alkalosis

b. Acute acidosis Rationale:In acidosis, ECF hydrogen ions shift into the cells in exchange for ICF potassium and sodium; hyperkalemia and acidosis therefore often occur together.

Which of the following buffer pairs is considered the major plasma buffering system? a. Protein/fat b. Carbonic acid/bicarbonate c. Sodium/potassium d. Amylase/albumin

b. Carbonic acid/bicarbonate Rationale: The carbonic acid/bicarbonate buffer pair operates in both the lung and the kidney and is a major extracellular buffer.

A 25-year-old male is diagnosed with a hormone-secreting tumor of the adrenal cortex. Which finding would the nurse expect to see in the lab results? a. Decreased blood volume b. Decreased blood K+ levels c. Increased urine Na+ levels d. Increased white blood cells

b. Decreased blood K+ levels Rationale: Aldosterone is secreted from the adrenal cortex. It promotes renal sodium and water reabsorption and excretion of potassium, leading to decreased potassium levels.

While planning care for elderly individuals, the nurse remembers the elderly are at a higher risk for developing dehydration because they have a(n): a. Higher total body water volume b. Decreased muscle mass c. Increase in thirst d. Increased tendency towards developing edema

b. Decreased muscle mass

A 52-year-old diabetic male presents to the ER with lethargy, confusion, and depressed reflexes. His wife indicates that he does not follow the prescribed diet and takes his medication sporadically. Lab results indicate hyperglycemia. Which assessment finding is most likely to occur? a. Clammy skin b. Decreased sodium c. Decreased urine formation d. Metabolic alkalosis

b. Decreased sodium Rationale: Hypertonic hyponatremia develops with hyperglycemia. Increases in plasma lipids displace water volume and decrease sodium concentration, leading to the symptoms described.

Hyperlipidemia and hyperglycemia are associated with: a. Hypernatremia b. Hypertonic hyponatremia c. Hypokalemia d. Acidosis

b. Hypertonic hyponatremia

For a patient experiencing metabolic acidosis, the body will compensate by: a. Excreting H+ through the kidneys b. Hyperventilating c. Retaining CO2 in the lungs d. Secreting aldosterone

b. Hyperventilating Rationale: In an attempt to compensate for metabolic acidosis, the lungs hyperventilate to blow off CO2. It is the lungs hyperventilating that would compensate for metabolic acidosis, not the kidneys. CO2 retention would increase the acidotic state. Aldosterone would conserve water, but does not help compensate for acidosis.

A nurse is teaching the staff about antidiuretic hormone (ADH). Which information should the nurse include? Secretion of ADH is stimulated by: a. Increased serum potassium b. Increased plasma osmolality c. Decreased renal blood flow d. Generalized edema

b. Increased plasma osmolality

Which of the following patients should the nurse assess for a decreased oncotic pressure in the capillaries? A patient with: a. A high-protein diet b. Liver failure c. Low blood pressure d. Low blood glucose

b. Liver failure Rationale: Liver failure leads to lost or diminished plasma albumin production, and this contributes to decreased plasma oncotic pressure.

A patient has been searching on the Internet about natriuretic hormones. When the patient asks the nurse what do these hormones do, how should the nurse respond? Natriuretic hormones affect the balance of: a. Calcium b. Sodium c. Magnesium d. Potassium

b. Sodium Rationale: Natriuretic hormones are sometimes called a "third factor" in sodium regulation.

Cerebral edema is an increase in the fluid content of the:

brain tissue.

When a woman's uterus is assessed as protruding through the entrance of the vagina to the hymen, which grade of prolapse does this indicate? a. 0 b. 1 c. 2 d. 3

c. 2

Which description is used when a progressive neoplastic change involves the full epithelial thickness of the cervix? a. Cervical intraepithelial neoplasia b. Cervical dysplasia c. Cervical carcinoma in situd. Invasive carcinoma of the cervix

c. Cervical carcinoma in situ

Which term is used to identify the descent of the posterior bladder and trigone into the vaginal canal? a. Rectocele b. Vaginocele c. Cystocele d. Enterocele

c. Cystocele

Which term is used to identify benign uterine tumors that develop from smooth muscle cells in the myometrium and are commonly called uterine fibroids? a. Endometrial polyps b. Myometrial polyps c. Leiomyomas d. Myometriomas

c. Leiomyomas

Fluid-filled squishy sacs characterize which breast disorder? a. Paget disease b. Cysts c. Nonproliferative breast lesions d. Lobular carcinoma in situ

c. Nonproliferative breast lesions

What statement concerning the pathogenetic mechanisms of polycystic ovarian syndrome (POS) is true? a. POS causes a decrease in leptin levels; this decrease reduces the hypothalamic pulsatility of gonadotropin-releasing hormone, which reduces the number of follicles that mature. b. POS is a result of a disorder in the anterior pituitary that increases the follicle-stimulating hormone, which reduces the luteinizing hormone released. c. POS is a result of a combination of conditions that include oligo-ovulation or anovulation, elevated levels of androgens, or clinical signs of hyperandrogenism and polycystic ovaries. d. POS inhibits testosterone, which stimulates androgen secretion by the ovarian stroma and indirectly reduces sex hormone-binding globulin.

c. POS is a result of a combination of conditions that include oligo-ovulation or anovulation, elevated levels of androgens, or clinical signs of hyperandrogenism and polycystic ovaries.

Considering the mediating factors of premenstrual syndrome (PMS), which medication may be used either continually or only during the menstrual period as a treatment for the condition? a. NSAIDs b. Estrogen c. SSRIs d. Progesterone

c. SSRIs

What are typical findings on breast palpation of a woman diagnosed with simple fibroadenoma? a. Painful, round, movable, and fluid-filled mass b. Painless, movable, hard, and irregular mass c. Smooth, solid, mobile, and well-circumscribed mass d. Smooth, nonmovable, irregular, and soft mass

c. Smooth, solid, mobile, and well-circumscribed mass

A nurse is discussing endocrine system dysfunction with a patient. Which statement indicates the patient understood? Endocrine system dysfunction can result from hyposecretion, hypersecretion, or from:

Abnormal receptor activity

Clinical manifestations that include irregular or heavy bleeding, the passage of large clots, and the depletion of iron stores support which diagnosis?

Abnormal uterine bleeding

Which of the following is FALSE regarding the sources of increased ammonia that contribute to hepatic encephalopathy?

Accumulation of short-chain fatty acids attaches to ammonia

Which of the following is false regarding the sources of increased ammonia that contribute to hepatic encephalopathy?

Accumulation of short-chain fatty acids attaches to ammonia

A 29-year-old female presents with cloudy urine, flank pain, hematuria, and fever. Which of the following does the nurse suspect the patient is most likely experiencing?

Acute cystitis

A 30-year-old male is demonstrating hematuria with red blood cell casts and proteinuria exceeding 3 to 5 g/day, with albumin being the major protein. The most probable diagnosis the nurse will see documented on the chart is:

Acute glomerulonephritis

A 19-year-old female was involved in a motor vehicle accident during which she sustained a closed head injury. She is now experiencing detrusor sphincter dyssynergia. Which of the following is the most beneficial medication treatment?

Alpha blocker

When a nurse observes poststreptococcal glomerulonephritis as a diagnosis on a patient, which principle will the nurse remember? Acute poststreptococcal glomerulonephritis is primarily caused by:

Antigen-antibody complex deposition in the glomerular capillaries and inflammatory damage

A healthcare professional is educating a community men's group on symptoms of benign prostatic hyperplasia (BPH). The professional relates that the most symptoms are a result of which pathophysiologic condition?

Compression of the urethra

In order to help prevent a preadolescent girl from developing later cervical cancer, which virus does the healthcare professional recommend vaccination against to the parent?

HPV

The primary cause of duodenal ulcers is:

Helicobacter pylori.

A 42-year-old male is involved in a motor vehicle accident during which he loses a lot of blood. The nurse realizes he is in acute renal failure caused by:

Inadequate renal blood flow

Reflux esophagitis may be defined as a(n):

Inflammatory response to gastroesophageal reflux

A person has been diagnosed with primary dysmenorrhea and wants to know why ibuprofen is a good choice for pain control. What response by the healthcare professional is best?

It reduces the production of prostaglandins in your body

While reviewing urine lab results, the nurse remembers the glomerular filtration rate (GFR) is directly related to the:

Perfusion pressure in the glomerular capillaries

A 54-year-old female is diagnosed with nephrotic syndrome. Which of the following is a common symptom of this disease?

Proteinuria

A 28-year-old female presents with fever, chills, and flank pain. She is diagnosed with pyelonephritis. A nurse recalls the patient's infection is located in the:

Renal pelvis

A student is learning about pelvic inflammatory disease (PID). What information does the student clarify with a study partner as being correct?

Such an inflammation results in permanent changes to the ciliated epithelium of the fallopian tubes

After a gastrectomy, individuals develop anemia from deficiencies in iron, folate, and vitamin B12.

True

Ammonia formation may increase hepatic encephalopathy

True

Ammonia formation may increase hepatic encephalopathy.

True

Anorexia is the lack of desire to eat despite physiologic stimuli that would normally produce hunger

True

Anorexia is the lack of desire to eat despite physiologic stimuli that would normally produce hunger.

True

Because patterns of bowel evacuation differ greatly among individuals, constipation must be individually defined.

True

Chronic gastritis tends to occur in older adults and causes thinning and degeneration of the stomach wall.

True

Endoscopy and biopsy may show long-standing inflammatory process and gastric atrophy indicating chronic gastritis in an individual with no history of abdominal distress.

True

Hepatitis B is a sexually transmitted disease

True

Hepatitis D virus (HDV) occurs in individuals with hepatitis B

True

Hepatitis D virus (HDV) occurs in individuals with hepatitis B.

True

Individuals with chronic hepatitis C are at increased risk for chronic liver disease

True

Individuals with chronic hepatitis C are at increased risk for chronic liver disease.

True

Gastroesophageal reflux disease (GERD) is a result of:

a zone of low pressure of the lower esophageal sphincter (LES).

Increased bilirubin production, impaired hepatic uptake and excretion of bilirubin, and reabsorption of bilirubin in the small intestine can each lead to:

physiologic jaundice of the newborn.


Kaugnay na mga set ng pag-aaral

American Government Ch 10 Interest Groups and Lobbying

View Set

Origins, Insertions, Innervations

View Set

NUR 2211 Medical Surgical Nursing Chapter 27: Nursing Management: Patients With Renal Disorders

View Set